GENERAL PRACTITIONER EXAM
Question Summary
0 of 100 questions completed
Questions:
- 1
- 2
- 3
- 4
- 5
- 6
- 7
- 8
- 9
- 10
- 11
- 12
- 13
- 14
- 15
- 16
- 17
- 18
- 19
- 20
- 21
- 22
- 23
- 24
- 25
- 26
- 27
- 28
- 29
- 30
- 31
- 32
- 33
- 34
- 35
- 36
- 37
- 38
- 39
- 40
- 41
- 42
- 43
- 44
- 45
- 46
- 47
- 48
- 49
- 50
- 51
- 52
- 53
- 54
- 55
- 56
- 57
- 58
- 59
- 60
- 61
- 62
- 63
- 64
- 65
- 66
- 67
- 68
- 69
- 70
- 71
- 72
- 73
- 74
- 75
- 76
- 77
- 78
- 79
- 80
- 81
- 82
- 83
- 84
- 85
- 86
- 87
- 88
- 89
- 90
- 91
- 92
- 93
- 94
- 95
- 96
- 97
- 98
- 99
- 100
Information
Hi, Welcome to General Practioner Exam
You have already completed the Exam before. Hence you can not start it again.
Exam is loading...
You must sign in or sign up to start the Exam.
You have to finish following Exam, to start this Exam:
Results
0 of 100 questions answered correctly
Your time:
Time has elapsed
You have reached 0 of 0 points, (0)
Average score |
|
Your score |
|
Categories
- Not categorized 0%
Pos. | Name | Entered on | Points | Result |
---|---|---|---|---|
Table is loading | ||||
No data available | ||||
- 1
- 2
- 3
- 4
- 5
- 6
- 7
- 8
- 9
- 10
- 11
- 12
- 13
- 14
- 15
- 16
- 17
- 18
- 19
- 20
- 21
- 22
- 23
- 24
- 25
- 26
- 27
- 28
- 29
- 30
- 31
- 32
- 33
- 34
- 35
- 36
- 37
- 38
- 39
- 40
- 41
- 42
- 43
- 44
- 45
- 46
- 47
- 48
- 49
- 50
- 51
- 52
- 53
- 54
- 55
- 56
- 57
- 58
- 59
- 60
- 61
- 62
- 63
- 64
- 65
- 66
- 67
- 68
- 69
- 70
- 71
- 72
- 73
- 74
- 75
- 76
- 77
- 78
- 79
- 80
- 81
- 82
- 83
- 84
- 85
- 86
- 87
- 88
- 89
- 90
- 91
- 92
- 93
- 94
- 95
- 96
- 97
- 98
- 99
- 100
- Unanswered
- Answered
- Review
-
Question 1 of 100
1. Question
1 pointsA 71 year old woman dies 3 days after ICU admission. Clinical picture is characterized by fever, purulent sputum, and pulmonary infiltrates on CXR. Autopsy shows patchy areas of consolidation in both lungs, with neutrophilic exudate filling bronchi and bronchioles on microscopy. Gram staining reveals colonies of small gram negative bacilli. The most likely diagnosis is
Correct
Incorrect
Explanation:
Hospital acquired pneumonia (or nosocomial pneumonia) is defined as a pneumonia that mates more than 48 hours after admission to the hospital. It more commonly affects patients who are in the ICU or are mechanically ventilated. The most frequent causative microorganisms are Pseudomonas aeruginosa, Staphylococcus aureus, Klebsiella pneumoniae, E.coli, and Enterobacter. Pathologic features are those of acute bronchopneumonia, but the mortality rate is high, around 50%. Anaerobic pneumonia occurs in individuals predisposed to aspiration, i.e., those with depressed levels of consciousness, impaired deglutition, or tracheal/nasogastric tubes. Periodontal disease is an additional risk factor, as it leads to increased numbers of anaerobic bacteria in aspirated material. Necrotizing pneumonia, lung abscess, and pleural empyema are the most common pathologic lesions. Community acquired pneumonia is usually caused by pneumococcus (Streptococcus pneumoniae) and results in homogeneous consolidation of an entire lobe. Microscopically, a fibrinopurulent exudate fills the alveolar spaces. Lipid pneumonia is characterized histologically by large numbers of lipid-laden macrophages. This form of pneumonia occurs in association with obstructive bronchial lesions or aspiration of mineral oils. In primary atypical pneumonia there is a lympho monocytic infiltrate confined to interalveolar septa and interstitium. Mycoplasma pneumoniae is the most frequent etiologic agent.
-
Question 2 of 100
2. Question
1 pointsMarrow aspirates from an adult with Hemoglobin SS disease (sickle cell anemia) and another with normal Hemoglobin A are being studied. Marrow aspirate from sickle cell anemia patient could be identified because of its increased
Correct
Incorrect
Explanation:
Adults with sickle cell disease have undergone decades of accelerated RBC formation and destruction, leading to accelerated erythropoiesis in the bone marrow. Consequently, the bone marrow becomes hyperplastic, with marked increases in the number of normoblasts (erythroblasts) at the expense of marrow fat and marrow bone. Although the white cell and megakaryocyte lines are undiminished, there is a marked increase in RBC precursors and iron stores. Iron storage increases as a consequence of both chronic transfusions and increased dietary absorption; these increased iron stores can be appreciated with a Prussian blue stain.
-
Question 3 of 100
3. Question
1 pointsCorrect statement regarding thrombocytosis is which one of the following?
Correct
Incorrect
Explanation:
The most common cause of thrombocytosis is a reactive thrombocytosis. Thrombocythaemia may occur in any of the myeloproliferative disorder particularly polycythaemia rubra vera (PRV). Thrombopoietin is the key hormone in the regulation of megakaryocyte differentiation. Secondary thrombocytosis does not place the patient at risk for haemostatic or cardiovascular events.
-
Question 4 of 100
4. Question
1 pointsTrue statement regarding of psittacosis (ornithosis) is which one of the following?
Correct
Incorrect
Explanation:
Chlamydia psittaci is endemic in birds including psittacine birds, canaries, finches, pigeons and poultry. Pet owners, vets and zoo keepers are most at risk. It is rare in children. Person to person transmission occurs especially in a hospital environment. Sputum Gram stain reveals a few leucocytes and no predominant bacteria. There are few signs and few laboratory/x ray findings. Positive serology is with complement fixing antibodies. It is treated with tetracycline.
-
Question 5 of 100
5. Question
1 pointsIn which one of the following conditions is deoxyribonucleic acid (DNA) analysis the most useful diagnostic test?
Correct
Incorrect
Explanation:
Klinefelter´s syndrome and Down syndrome are diagnosed principally by chromosomal analysis/karyotype – XXY in the former and trisomy 21 or translocation in the latter. A trinucleotide CAG repeat expansion in the Huntington gene is diagnostic of Huntington disease.
The majority of cases of hypertrophic cardiomyopathy (HOCM) are autosomal dominantly inherited yet defective genes are located on a variety of chromosomes.
DNA linkage analysis is used to assist in the diagnosis of adult polycystic kidney disease (PCKD) but the presence of multiple copies continues to complicate the development of reagents for direct genetic testing, at least of the 70% of PKD1 that is replicated elsewhere. -
Question 6 of 100
6. Question
1 pointsA 19-year-old Arabian female with α1-antitrypsin deficiency attends clinic 22 weeks pregnant. She wants to know if her child will be affected. What is the mode of inheritance of this disease?
Correct
Incorrect
Explanation:
α1-antitrypsin deficiency is inherited in an autosomal recessive pattern, which causes emphysema and cirrhosis and has a prevalence of 1 in 2500
-
Question 7 of 100
7. Question
1 pointsAn 18-year-old man collapsed shortly after being stung on the hand by a bee. On examination, his blood pressure was 80/40 mmHg, and facial swelling was noted. Which one of the following investigations is most likely to confirm the nature of the reaction?
Correct
Incorrect
Explanation:
Type I hypersensitivity, also known as immediate or anaphylactic hypersensitivity, usually takes 15 – 30 minutes from the time of exposure to the antigen. The reaction may cause a range of symptoms from minor inconvenience to death. The reaction involves preferential production of IgE, in response to certain antigens, which in turn initiates a sequence of event: leading to the release various pharmacologically active substances that are responsible for the clinical features. Diagnostic tests include skin tests, measurement of total IgE and specific IgE antibodies against the suspected allergens. However, this question asks which of the following tests would provide confirmatory information and that would be Tryptase. Tryptase is a neutral protease stored in mast cell secretory granules that is secreted by human mast cells. Levels in normal blood are undetectable (< l ng/ml). Elevated serum levels demonstrate that mast cell activation with mediator release has occurred whether triggered IgE-mediated anaphylaxis or non-IgE-mediated anaphylactoid reactions. The greater the severity of anaphylaxis, the more likely that serum -tryptase levels will be elevated
-
Question 8 of 100
8. Question
1 pointsWhich one of the following is highly specific for Wegner´s granulomatosis?
Correct
Incorrect
Explanation:
When requesting an ANCA test, both immunofluorescence and an ELISA test are generally performed. On immunofluoresecence, if ANCA are present the staining pattern; the staining pattern may be cytoplasmic (cANCA) or perinuclear (pANCA). Typical antigen specificity includes proteinase 3 or myeloperoxidase. cANCA and specificity for the PR-3 antigen is most specific for Wegener´s granulomatosis. This pattern is also seen in microscopic polyarteritis nodosa and rarely Churg-Strauss syndrome. PANCA and MPO are less specific findings detected in various vasculitic illnesses and occasionally in chronic infections.
-
Question 9 of 100
9. Question
1 pointsAll of the following are true regarding adipocytes, EXCEPT?
Correct
Incorrect
Explanation:
Adipocytes cease growth after they reach a volume of approximately one microgram (C). Adipocytes (fat cells) form a reservoir of energy that expands or contracts, depending on the energy balance of the organism (A). These cells develop from precursor preadipocytes to accommodate excess nutrient calories (B). New adipocytes are formed ad infinitum in response to continuing positive energy balance (D). Despite subsequent weight loss, the fat cell number remains constant (E).
-
Question 10 of 100
10. Question
1 pointsOvarian carcinoma that is associated with an elevated alpha-fetoprotein (AFP) level is
Correct
Incorrect
Explanation:
CA-125 may be elevated in ovarian epithelial tumors, such as a cystadenocarcinoma. AFP can be elevated with either an endodermal sinus tumor or embryonal carcinoma. An elevated hCG can be seen with choriocarcinomas. Sertoli-Leydig and granulosa-theca cell tumors do not usually produce an elevated CA- 125, AFP, or hCG level;
-
Question 11 of 100
11. Question
1 pointsThe female climacteric is NOT associated with which of the following?
Correct
Incorrect
Explanation:
Decreased size of the female reproductive organs and breasts is a direct consequence of estrogen deficiency. Menopause is defined as the final episode of menstrual bleeding in women. The climacteric is the physiologic period during which ovarian function regresses. The menstrual cycle shortens due to a shortened follicular phase, with increased FSH, normal LH, and decreased E2 and progesterone, when compared to normal ovulatory cycles. Vasomotor instability typified by the hot flash, is accompanied by decreased skin resistance, peripheral vasodilatation, and LH pulses. Development of osteoporosis is affected by many factors, including diet, smoking, and estrogen deprivation.
-
Question 12 of 100
12. Question
1 pointsWhich of the following is NOT correct association between molecular genetic abnormalities and colon adenomas/carcinomas Correct
Incorrect
Explanation:
As currently understood, a series of genetic changes occur in concert to initially produce an adenoma that may subsequently progress to a carcinoma. The initial step probably involves a mutation of the K-ras proto-oncogene. The gene for familial adenomatous polyposis has been mapped to chromosome 5. Chromosome 17(p53 gene) and chromosome 18 (DDC gene) deletions are thought to be important in malignant transformation. Overexpression of the c-myc gene has been reported in colon cancers.
-
Question 13 of 100
13. Question
1 pointsWhich of the following statement is FALSE regarding osteomyelitis?
Correct
Incorrect
Explanation:
Although 70% of compound fractures are contaminated with bacteria, less than 10% result in infection because of extensive operative debridement and perioperative antibiotic therapy. The anatomic location of hematogenous osteomyelitis depends on the age of the affected person. In children, hematogenous osteomyelitis usually affects the metaphysis of the long bones due to its large blood flow. In adults, the vertebrae are more vascular than other skeletal tissues. Therefore, bacteremias seed vertebral bodies preferentially at the more vascular anterior end plates. In up to two thirds of diabetics, osteomyelitis may develop by contiguous spread from an infected foot ulcer. Indwelling foreign bodies, such as prosthetic joints, decrease the quantity of bacteria necessary to establish a bone infection. These devices also permit pathogens to persist on their surface, protected from circulating immune factors and systemic antibiotics.
-
Question 14 of 100
14. Question
1 pointsWhich one of the following is NOT included in macrophage secretions?
Correct
Incorrect
Explanation:
Macrophages are long lived phagocytic cells important in the inflammatory response. Some of the many secretory products of macrophages include binding proteins such as fibronectin, enzyme inhibitors such as alpha-2 macroglobulins, pyrogens such as interleukin I, and tumor necrosis factor, also known as cachectin. Exotoxin is secreted by bacteria
-
Question 15 of 100
15. Question
1 pointsWhich one of the following is NOT a surface component and extracellular product produced by Group A streptococci?
Correct
Incorrect
Explanation:
Surface components and extracellular products produced by Group A streptococci are important in the development of infection. The M protein is the major surface protein of Group A streptococci. This protein acts as a membrane anchor and correlates with the capacity of a strain to resist phagocytic killing. The extracellular enzyme, hyaluronidase, facilitates the spread of infection along fascial planes by hydrolyzing hyaluronic acid in deeper tissues. Pyrogenic exotoxins types A, B, and C induce lymphocyte blastogenesis, potentiate endotoxin-induced shock, induce fever, and suppress antibody synthesis. Streptolysins O and S damage cell membranes and account for the hemolysis produced by the bacterium. CAMP factor is a phospholipase produced by Group B streptococci, which causes hemolysis synergistically with beta-lysins.
-
Question 16 of 100
16. Question
1 pointsA male aged 58 years is diagnosed with Wegener´s granulomatosis. LEAST likely finding in this patient is which one of the following?
Correct
Incorrect
Explanation:
Wegener´s granulomatosis is an immune disorder associated with abnormal production of c-ANCA autoantibodies. The vasculitis typically affects small and medium sized vessels. However, there is often an accompanying necrotizing glomerulonephritis and necrotizing granuloma formation in the upper respiratory tract. The therapy for Wegener´s granulomatosis is immunosuppression. “Palpable purpura” is associated with hypersensitivity angiitis.
-
Question 17 of 100
17. Question
1 pointsThe rationale for giving prophylactic antibiotics at the start of a surgery can be thought of as an attempt to prevent the bacteria from entering which of the following growth phases? Correct
Incorrect
Explanation:
A single dose of a prophylactic antibiotic given as surge is about to begin (sometimes given intravenously to assure the timing) has become more common because it is associated with a decreased wound infection rate. Such antibiotic usage does not prevent organisms from entering the tissues, but effectively prevents them from becoming established as they try to grow and divide (e.g., leave the lag phase and enter the log or exponential phase of colony growth). The stationary phase and phase of decline (formerly called death phase) would occur much later, after a colony had been established (which is what the surgeons were trying to prevent).
-
Question 18 of 100
18. Question
1 pointsType and cross match tests of a patient in emergency reveal the following:
Patients RBC plus serum from A person Agglutination Patients RBC plus serum from B person Agglutination Patients RBC plus serum from AB person No agglutination Patients RBC plus serum from O person Agglutination Patient´s RBC plus anti RhD serum Agglutination Patients serum plus type A RBC No agglutination Patients serum plus type B RBC No agglutination Patients serum plus type AB RBC No agglutination Patient´s serum plus type O RBC No agglutination Patients serum plus RhD RBC No agglutination The physician should order what blood type to match this patient´s allotype?
Correct
Incorrect
Explanation:
The ABO blood group allotypes reflect the presence of glycoproteins on the erythrocyte surface. Humans make IgM isohemagglutinins naturally against similar glycoproteins found on the surface of normal flora organisms, but self-tolerance should prevent the production of antibodies that would agglutinate the person´s own erythrocytes. This patient is AB, Rh+. Persons with this blood type will not possess isohemagglutinins in their bloodstream against either A or B antigens. Their PBCO will agglutinate in the presence of sera from individuals with type A, type B or type C blood. The Rh factor is either present (+) or absent (-) from red cells. Since the patient´s cells agglutinated in the presence of anti-Rh serum, it is clear that the patient is ´RhD+.
In emergencies, it is always a convenient short cut to use blood that is C, RhD-, the “universal donor,” since it has none of the alloantigens that could be recognized on the surfaces of cells. The question, however, inquires which type of blood would match the patient, not which could be used in case of emergency. -
Question 19 of 100
19. Question
1 pointsA 7 month old child is hospitalized for a yeast infection that does not respond to therapy. He has a history of multiple acute pyogenic infections. On examination the spleen and lymph nodes are not palpable. A differential WBC count shows 95 neutrophils, 1 lymphocyte, and 4 monocytes. Bone marrow biopsy contains no plasma cells or lymphocytes. CXR reveals the absence of a thymic shadow. Tonsils are absent. The most common cause of this condition is
Correct
Incorrect
Explanation:
Severe combined immunodeficiency (SCID) is associated with deficiencies in both Band T cells due to a defect in differentiation of an early stem cell. Over 50 % of the cases are caused by a gene defect on the X chromosome, resulting in a defective IL-2 receptor. Adenosine deaminase deficiency is an autosomal recessive disease characterized by a deficiency of adenosine deaminase, which results in accumulation of metabolites that are toxic to both B and T stem cells in the bone marrow. Children with severe infections usually die within the first 2 years of life unless they receive bone-marrow transplants. A defective CD40L molecule causes hyper-IgM syndrome. This makes it impossible for B lymphocytes to receive the signal to switch isotypes, and they are therefore “stuck” making IgM alone. It is not classified as a SCID. Inability to produce MHC-II causes bare lymphocyte syndrome, which causes patients to be unable to produce CD4 cells. It results in a type of SCID since TH cells are necessary for the function of all immune responses. Nonsense mutation of rag genes results in a SCID because without the ability to perform VDJ recombinations, lymphocytes cannot express surface receptors for antigen; and in that case, lymphocyte precursors are induced to undergo apoptosis in the bone marrow. Recombination activating genes (rag genes) are responsible for the VDJ rearrangements that allow lymphocytes to create the idiotype of their antigen receptors.
-
Question 20 of 100
20. Question
1 pointsA 15 month old child having recurrent skin, ear and respiratory infections is diagnosed with common variable immunodeficiency. Finding that is most likely to have contributed to this diagnosis, as opposed to a diagnosis of X-linked hypogammaglobulinemia is
Correct
Incorrect
Explanation:
Common variable immunodeficiency is a disease caused by abnormal B cell differentiation, characterized by decreased numbers of plasma cells and inadequate immunoglobulin production of all classes. It is distinguished from Bruton X-linked hypogammaglobulinemia by the fact that the number of B lymphocytes in the blood and lymphoid organs will be near normal. In Bruton agammaglobulinemia, B-cell maturation stops after the rearrangement of heavy-chain genes, and therefore, B cells in the bloodstream and lymphoid organs will be decreased in number or absent. Absence of plasma cells would be expected with both common variable immunodeficiency and Bruton hypogammaglobulinemia. Normal T-lymphocyte function would be expected with both common variable immunodeficiency and Bruton hypogammaglobulinemia. Rudimentary germinal centers would be found in Bruton hypogammaglobulinemia but not in common variable immunodeficiency. Susceptibility to mucosal surface pathogens would be observed with both common variable immunodeficiency and Bruton hypogammaglobulinemia.
-
Question 21 of 100
21. Question
1 pointsA 4 year old boy has suffered repeated viral infections and a serious infection following inoculation of the MMR vaccine. Studies using flow cytometry show that the child has normal numbers of CD19+, CD16+, and CD4+ cells but a near total absence of CD8+ cells. What molecule is likely to be missing from this child´s cells?
Correct
Incorrect
Explanation:
This child has MHC class I deficiency, which causes detect in the development of CD8+cells in the thymus. Because there is no MHC class l, cytotoxic cells cannot be selected, although helper cells (CD4+), which recognize MHC class II molecules, are normally produced.
The gamma-delta T-cell receptor is present on a small proportion of T cells that do not undergo thymic selection. Their function in the body is not known, but they would not be affected by the absence of the class I MHC.
Intercellular adhesion molecule-1 is a molecule found on the surface of antigen- presenting cells in the body that reacts with LEA-1 on T cells for the purpose of cell-to-cell adhesion. Its absence would not affect the development of cells in the thymus. The killer inhibitory receptor is one of two receptors found on the surface of natural killer cells. It inhibits the killing of an attached cell if it is bound to MHC class I. Because this child has normal levels of CD16+ (natural killer) cells, it is unlikely he has a deficiency of this receptor. MHC class II is the molecule that presents peptides that have been processed in intracellular vesicles by antigen presenting cells to T-helper cells. If this molecule were absent from the child, he would be suffering from bare lymphocyte syndrome, which is associated with a failure of education of CD4+ helper cells, not CD8+ cells as described here. -
Question 22 of 100
22. Question
1 pointsA research lab is studying mechanisms of immunity to Babesia microti in mice. From immune mice the cells are separated by flow cytometry, using fluorescently labeled antibodies directed against a variety of cell-surface markers. Infected RBCs are then added to the separated populations of immune cells and observed for lysis in the absence of complement. The cell surface marker that would be most useful in identifying a cell capable of lysing infected cells in this system is
Correct
Incorrect
Explanation:
Babesia microti is an intraerythrocytic parasite that causes a mild anemia and fever in affected humans. It is transmitted by the same tick that transmits Lyme disease Ixodes scapularis. Red blood cells are the only cells in the body that are devoid of MHC class I molecules so lysis of these cells by a cytotoxic cell would only be possible using NK cells.
NK cells are the cytotoxic cell that kills in the absence of MHC class I recognition. NK cells are actively inhibited if they recognize these antigens on a target cell. The most specific marker for the identification of NK cells is CD56, although CD16 is also found on their surface. CD4 cells are helper T cells. They assist in both antibody-mediated and cell-mediated immune responses, but by themselves, as stipulated in this question stem, they would not be capable of lysis of red blood cells. CD8 cells are cytotoxic T cells. These cells kill by identifying alterations in the MHC class I molecule. Since such molecules are absent from erythrocytes, cytotoxic T cells are not capable of lysing erythrocytes. CD14 is the endotoxin receptor that is found on macrophages. Macrophages are capable of ingesting and destroying infected erythrocytes in the spleen, but are not capable of causing their lysis in this system. CD16 is the receptor for the Fc component of immunoglobulin G. It is present on a variety of cells, including natural killers, macrophages, and neutrophils. Therefore, it is not the best marker for the NK cell uniquely but would label several populations of cells. -
Question 23 of 100
23. Question
1 pointsRoutine examination of a 54 year old male shows fusiform enlargement of the distal fingers, with upward doming of the nail beds. He has long standing emphysema for which he requires oxygen and bronchodilatory therapy. Hand radiograph shows new bone formation beneath the periosteum of the distal phalanges. This is most strongly associated with which disorder? Correct
Incorrect
Explanation:
Enlargement of the distal segments of the fingers and toes due to proliferation of connective tissue is known as clubbing. Clubbing maybe associated with lung cancer, mesothelioma, bronchiectasis, and hepatic cirrhosis. In these conditions, the clubbing is accompanied by formation of new subperiosteal bone (hypertrophic osteoarthropathy or hypertrophic pulmonary osteoarthropathy-HPO). Additionally, this patient has emphysema likely due to smoking, which also causes lung cancer. It is now the subject of vigorous debate whether it is the cancer that actually causes HPO or the emphysema, which is often associated with lung cancer. Either way, when HPO is identified, it is incumbent upon the clinician to look for a potential lung cancer. Chronic renal failure causes renal osteodystrophy, which may manifest s combination of osteitis fibrosa cystica and osteomalacia. This is due to elevated parathyroid hormone, which causes the leaching of bones. Multiple exostoses are sometimes associated with multiple colonic polyps and colon cancer in Gardner syndrome. A variety of endocrine adenomas can be seen in polyostotic fibrous dysplasia. The most well known of these is McCune-Albright syndrome: precocious puberty and polyostotic fibrous dysplasia. Profound anemia can be seen in diseases that destroy the marrow, such as ac osteopetrosis, mastocytosis, myelofibrosis, lymphoma, or leukemia.
-
Question 24 of 100
24. Question
1 pointsA 4 year old girl´s physical examination is remarkable for extremity edema. Urinalysis shows significant proteinuria our no red cells or casts. Electron microscopy performed on a kidney biopsy would most likely reveal which finding?
Correct
Incorrect
Explanation:
The most frequent cause of nephrotic syndrome in children is minimal change disease (Iipoid nephrosis), which is characterized by an absence of findings by light microscopy and by fusion of epithelial toot processes on electron microscopy. Dense deposits are seen in type II membranoproliferative glomerulonephritis. Mesangial deposits are a prominent feature of IgA nephropathy and are seen to a lesser degree in several other glomerulonephritides. Subendothelial deposits are seen in type I membranoproliferative glomerulonephritis. Subepithelial spikes are a feature of membranous glomerulonephritis.
-
Question 25 of 100
25. Question
1 pointsA 54 year old smoker has severe, crushing, substernal chest pain. Coronary arteries are imaged. A complete occlusion of the left anterior descending coronary artery is observed. What best describes the pathophysiologic processes that produced his symptoms?
Correct
Incorrect
Explanation:
Although many of the events that produce atherosclerotic plaques remain elusive, several steps in the process are known. The first gross findings associated with atherosclerotic plaque formation are fatty streaks. Fatly streaks are found in the vasculature of most people in the second decade of life. The fatty streak is a focal accumulation of serum lipoproteins within the intima of the blood vessel wall. As smooth muscle cells migrate to the lesion they produce a connective tissue matrix that overlies the lipid-laden foam cells. When this fibrous covering dislodges, the thrombogenic contents of the lesion are exposed to the circulation and the ensuing clot then occludes the vessels. Tissue ischemia and infarction then occur. The fibrous lesions that follow the fatty streak seldom produce infarction through direct extension of the lesion ischemia may occur with resulting angina, but because the lesion growth is slow, collateral vessels may grow and thus prevent infarction. The fatly streak precedes fibrous lesions found in coronary artery disease. Lipid-laden macrophages, called foam cells, contribute to the core of atherosclerotic lesions. They do not, however, release their lipids through exocytosis, forming a lipid-based occlusion.
-
Question 26 of 100
26. Question
1 pointsA boy aged 5 years, hospitalized for cellulitis, exhibits global denudation of the skin resulting from splitting of the epidermis at the stratum granulosum. The most likely diagnosis is which one of the following?
Correct
Incorrect
Explanation:
Scalded skin syndrome is a pediatric condition caused by an exfoliative toxin produced by Staphylococcus aureus (which is a common cause of cellulitis). The toxin splits the epidermis at the level of the stratum granulosum, causing global denudation of the skin. Scalded skin syndrome is not associated with suntanning or sunburn. Impetigo is a superficial skin infection, usually caused by group A beta-hemolytic streptococci or staphylococci. Classic clues are eroded pustules covered by honey colored crusts. Impetigo may lead to post-streptococcal glomerulonephritis, a sign of which is red cell casts in the urine. Melasma consists of irregular patches of hyperpigmentation on the face. It most commonly appears during pregnancy and may not completely regress. Tinea corporis is a fungal infection. It is also known as “ringworm” because it presents as an expanding round lesion with an erythematous circinate border. Vitiligo is characterized by irregular patchy depigmentation of the skin that exhibits melanocyte deficiency microscopically. It is possibly autoimmune in origin and maybe related to stress. It is not associated with sun exposure and is a chronic condition.
-
Question 27 of 100
27. Question
1 pointsA 21 year old man with fatigue, recurrent fever, and enlarged cervical lymph nodes has numerous atypical lymphocytes in his peripheral blood smear. Biopsy from his enlarged node shows expansion of lymphoid follicles with preservation of the underlying architecture. Numerous atypical lymphocytes are present in the paracortical areas. What is the likely diagnosis?
Correct
Incorrect
Explanation:
Infectious mononucleosis is a benign infection caused by the Epstein Barr virus (EBV), a herpesvirus. Although B lymphocytes are infected by the virus, the characteristic atypical cells are activated cytotoxic and suppressor T cells-thus the paracortical location (T cell zone) in the lymph node. Lymph nodes in viral infections show expansion of germinal centers without loss of normal architecture. All lymphomas, including Burkitt, Hodgkin, and non Hodgkin lymphomas, destroy the normal architecture of the lymph node. AIDS is associated with a number of neoplastic and infectious processes that may alter the lymph node structure. The characteristic lymph node changes in AIDS are progressive transformation of the germinal centers, not paracortical hyperplasia. Burkitt lymphoma produces a sea of monotonous, mitotically active cells in a “starry sky” appearance. Hodgkin lymphoma also can show nodular or diffuse patterns but is characterized by the presence of Reed-Sternberg cells. Other non Hodgkin lymphomas show either a nodular appearance or diffuse sheets of cells that replace the germinal centers.
-
Question 28 of 100
28. Question
1 pointsDeficiency of which one of the following trace elements is implicated as a cause of cardiomyopathy?
Correct
Incorrect
Explanation:
Selenium deficiency is one of the reversible causes of dilated cardiomyopathy. -
Question 29 of 100
29. Question
1 pointsA 36-year-old female from Brazil presents to her physician with complaints of palpitations, shortness of breath, and orthopnea for several weeks. She also has symptoms of bloating, heartburn, and dysphagia. There is no associated fever or chills. Her chest X-ray reveals an enlarged heart. What is the diagnostic test of choice to make the diagnosis?
Correct
Incorrect
Explanation:
This patient has Chagas´ disease, which is endemic to Central American countries.
Infection with Trypanosoma cruzi is transmitted via reduviid bugs and can cause myocarditis, leading to congestive heart failure in its chronic stages.
Diagnosis of acute Chagas´ disease is by examination of the peripheral smear. Diagnosis of chronic Chagas´ disease is by immunofluorescence testing and ELISA. Echocardiogram would only tell us that the patient has cardiomegaly and a low ejection fraction, but not the cause of congestive heart failure. The barium swallow would diagnose achalasia or dilated esophagus secondary to Chagas´ disease. This would cause symptoms of dysphagia. Peripheral smear examination is useful to diagnose acute Chagas´ disease. Acute phase is generally limited to facial edema and nodule (chagoma) near the bite coupled with fever, lymphadenopathy, and hepatosplenomegaly. Cardiac manifestations are associated with the chronic form of the disease in which few trypomastigotes are found in the blood. The electrocardiogram may reveal conduction defects, but the specific diagnosis of Chagas´ disease cannot be made from the electrocardiogram. -
Question 30 of 100
30. Question
1 pointsWhich of the following is NOT included in the HACEK group?
Correct
Incorrect
Explanation:
Eikenella corrodens is the other member of the HACEK group (the “E” in HACEK), which comprises a group of fastidious Gram- negative bacteria. Enterococcus is not part of the group. HACEK organisms can cause a subacute bacterial endocarditis. IV drug abusers and individuals with prosthetic valves are more at risk for HACEK infection. -
Question 31 of 100
31. Question
1 pointsOut of the following, which ovarian carcinoma is associated with an elevated human chorionic gonadotropin (hCG) level?
Correct
Incorrect
Explanation:
CA-125 may be elevated in ovarian epithelial tumors, such as a cystadenocarcinoma. AFP can be elevated with either an endodermal sinus tumor or embryonal carcinoma. An elevated hCG can be seen with choriocarcinomas. Sertoli-Leydig and granulosa-theca cell tumors do not usually produce an elevated CA- 125, AFP, or hCG level. -
Question 32 of 100
32. Question
1 pointsObese adults with type II diabetes mellitus often have which of the following?
Correct
Incorrect
Explanation:
So called “adult onset” diabetes in overweight adults is often associated with increased insulin resistance. Such patients are not especially prone to diabetic ketoacidosis or diarrhea. Estrogen receptors have not been implicated in diabetes. -
Question 33 of 100
33. Question
1 pointsWhat is the number one cause of endocarditis in intravenous drug abusers?
Correct
Incorrect
Explanation:
Staphylococcus aureus is isolated in 60% of the cases of endocarditis from intravenous drug abuse (IVDA). Streptococci, enterococci, Gram negative bacilli, and fungi are also significant causes.
-
Question 34 of 100
34. Question
1 pointsMelanoma is most commonly associated with which one of the following?
Correct
Incorrect
Explanation:
Melanoma is most commonly associated with which one of the following?
-
Question 35 of 100
35. Question
1 pointsA 6 month old infant aged is brought to the ER because of trouble breathing. His mother was breastfeeding the child when she noticed that he seemed to be unable to suckle strongly, his eyelids were drooping, and his breaths were shallow and irregular. The infant is immediately placed on respiratory support. The mother added honey to the infant´s first solid foods.The genetic coding for this toxin is
Correct
Incorrect
Explanation:
This child has floppy baby syndrome, caused by ingestion of the spores of Clostridium botulinum germination of the organism in the gut of the infant and subsequent production of the botulinum toxin, which causes a flaccid paralysis by blocking release of acetylcholine in cholinergic synapses. The genetic coding for production of this neurotoxin is found in the genome of a lysogenic phage, and therefore is produced only in virus infected bacterial cells. Other medically important attributes that are encoded in this way include the O antigen of Salmonella the erythrogenic exotoxins of Streptococcus pyogenes and the diphtheria toxin. The bacterial chromosome is the location of all essential genes in the bacterium. Loss of any essential gene will result in the death of the organism. Although most exotoxins are encoded on bacterial plasmids, endotoxin production and the cholera toxin are chromosomally encoded. An episome is a plasmid, which has become stably integrated into the bacterial chromosome. This is the mechanism by which Hfr cells arise in nature, but is not known to play a role in exotoxin production in bacteria. Plasmids are extrachromosomal circles of DNA that replicate autonomously and carry genes for most exotoxins, fertility factors, and most antibiotic resistances. The botulinum toxin, the erythrogenic exotoxins of Streptococcus pyogenes, and the diphtheria toxin are not encoded on plasmids, but in the genomes of lysogenic phages. Transposons are mobile segments of DNA, which are capable of mediating their own movement with in and between strands of DNA inside cells. Transposition is believed to be responsible for the arising of multiple drug resistance plasmids, but is not believed to play a role in exotoxin production.
-
Question 36 of 100
36. Question
1 pointsA 16 year old girl has a painfully enlarged lymph node in her right axilla. Peripheral blood counts are within normal limits. Lymph node biopsy shows numerous granulomas filled with necrotic debris. The most likely infectious agent involved is
Correct
Incorrect
Explanation:
Bartonella henselae is a gram negative rod. Cats and dogs are the reservoir transmitting the organism through bites or scratches. It causes cat scratch fever which presents as regional lymphadenopathy with or without low grade fever and headaches. In the immunocompromised patient, it can cause bacillary angiomatosis, which is the vascular proliferative form of infection and can affect any organ system; although it most commonly affects the skin and subcutaneous tissue. Treatment is with azithromycin or doxycycline. Borrelia burgdorferi is a spirochetal organism that is transmitted by a tick bite (Ixodes spp), producing Lyme disease. Lyme disease progresses from a skin rash to fevers, headache, and pain for about a month. It may produce lymphadenopathy, but is not associated with granuloma formation. Chlamydia psittaci infection occurs after contact with infected bird droppings and produces an atypical pneumonia. The central nervous system may also be involved, but lymph nodes are spared. Chlamydia trachomatis the chlamydial species that typically produces suppurative nodal granulomas (lymphogranuloma venereum). Coxiella burnetii infection is transmitted by inhaling dust or drinking milk from infected mammals, especially sheep and cows. The disease in humans, Q fever, is marked by mild nonspecific symptoms or pneumonia, and may progress to myocarditis or hepatitis. Rickettsia prowazekii produces epidemic (louse borne) typhus, which is transmitted by body lice and produces a rash akin to Rocky Mountain spotted fever. Although the organism (may reside in the lymph nodes in dormancy, it does not elicit granuloma formation. -
Question 37 of 100
37. Question
1 pointsA 4 year old boy presents with epistaxis. He has a history of multiple bacterial and viral respiratory tract infections and eczema. His uncle had similar problems. Examination is remarkable for multiple petechial lesions on the skin and mucous membranes. Serum IgE is increased and platelets are decreased. Cytokine that is most likely to be elevated in this patient is
Correct
Incorrect
Explanation:
This patient has Wiskott Aldrich syndrome, which is an X-linked condition characterized by eczema, thrombocytopenia, and repeated infections. Affected children may present with bleeding and often succumb to complications of bleeding infection, or lymphoreticular malignancy. The platelets are small, have a shortened half life, and appear to be deficient in surface sialophorin (CD43). Splenectomy can correct the thrombocytopenia but not the immune defect. Serum IgM is usually decreased, whereas lgE is frequently increased. Mutations in the Wiskott-Aldrich serum protein (WASP) gene on the short arm of the X chromosome are responsible for this disease. The TH2 cytokine that is most closely linked with isotype switching to promote IgE synthesis is IL-4. IL-1 is also known as endogenous pyrogen. It is a product of macrophages and it acts to set the hypothalamic temperature set point. IL-2 is a cytokine produced by all T-helper lymphocytes that causes cell division in virtually all lymphocytes. IL-3 is a product of many cells that stimulates bone- marrow production of myeloid lineage cells. IL-5 is a TH2 cytokine that stimulates B cells to isotype-switch to making IgA. -
Question 38 of 100
38. Question
1 pointsA patient is doing well on combination HIV therapy for the past five years. However, over the past several months, he has developed palpable axillary, cervical and inguinal lymph nodes. He now presents with oral lesions consistent with thrush. The most likely range for his CD4 cell count in cells/mm^3 at this time is
Correct
Incorrect
Explanation:
The patient has progressed from being asymptomatic to the point that he has a condition attributed to HIV infection, but not an AIDS defining condition. The most likely CD4 count associated with this stage of disease would be between 200 and 400 cells per mm^3 0-199 would be the expected CD4 cell count of a patient who has progressed to full blown AIDS. Full-blown AIDS is defined by the presence of a variety of infections or conditions that demonstrate a life-threatening collapse of the immune system. Examples of such conditions would include HIV encephalopathy, Pneumocystis pneumonia, Kaposi sarcoma, cryptosporidiosis, or Mycobacterium avium-intracellulare infections.
400-599 is a range of CD4 cell counts that would be associated with a finding of generalized lymphadenopathy or asymptomatic infection. The existence of thrush in the patient described indicates that he has progressed to the early symptomatic phase of the disease. 600-799 is a range of CD4 cell counts that would be expected in an asymptomatic patient, without generalized lymphadenopathy or thrush. Above 800 is considered a normal value for CD4 cells in an uninfected individual. -
Question 39 of 100
39. Question
1 pointsAn 8 year old boy´s blood is analyzed by flow cytometry. Physician wishes to determine the relative proportion of all cells bearing surface immunoglobulin in the circulation. Cell surface marker that should be used to identify the B cells in this blood sample is
Correct
Incorrect
Explanation:
The best markers for identification of B cells are CD19, CD20, and CD21. The CD21 marker is a receptor for EBV (Epstein – Barr virus). The CD3 marker is present on all T cells with either a CD4 or CD8 marker. This is the marker that is used to identify total T cell count in a blood sample. The CD3 marker is used for signal transduction in the different T cells. The CD4 marker is used to identify T helper cells. These are the cells that recognize exogenous peptides presented on MHC class II molecules by macrophages. CD4+ T helper cells are also involved in cell-mediated delayed hypersensitivity, production of cytokines for stimulation of antibody production by B cells, and stimulation of macrophages. The CD8 marker is used to identity cytotoxic T cells. These are the cells that recognize viral epitopes attached to the MHO class I molecules of a virally infected cell. The CD56 marker is used to identify NK (natural killer) cells. These cells are important in innate host defense, specializing in killing virally infected cells and tumor cells by secreting granzymes and perforins.
-
Question 40 of 100
40. Question
1 pointsA 3 month old boy is brought to the physician by his parents because of a “white dot” on his left eye that they noticed in pictures. Examination reveals a white pupillary reflex in the left eye. Orbit CT scan reveals a tumor in the left eye that does not involve the optic nerve. This condition is associated with an abnormality in which chromosomes?
Correct
Incorrect
Explanation:
About 20% of patients with a chromosome 13 abnormality (13q-syndrome) develop retinoblastoma. There is also a genetic dominant form of retinoblastoma that has an 80% penetrance rate. Retinoblastomas that have a genetic basis are more likely to be bilateral than the spontaneous lesions. Microscopically, retinoblastomas are composed of masses of small hyperchromatic cells that may form small rosettes composed of radially arranged cells surrounding a central lumen. -
Question 41 of 100
41. Question
1 pointsA 15 year old girl has a sudden onset of watery diarrhea tinged with blood. She was previously healthy. Two weeks ago she started a treatment for acne vulgaris with topical benzoyl peroxide and an oral antibiotic with a similar mechanism of action to macrolides that is mainly used to treat anaerobic infections. Examination reveals a slightly distended abdomen that is diffusely tender. Her temperature is 38.1°C (100.5°F). What is the most likely diagnosis?
Correct
Incorrect
Explanation:
Pseudomembranous enterocolitis is caused by the toxins produced by Clostridium difficile. Even if it is difficult to determine the exact antibiotic this patient is currently taking the report of a recently started therapy should instantly make you think of antibiotic associated diarrhea or pseudomembranous colitis. The oral antibiotic this patient is currently taking is clindamycin, which is an antibiotic commonly used (in addition to tetracyclines) for the treatment of moderate to severe acne. As stated in the question, it has a similar mechanism of action and resistance as macrolides and is generally used to treat infections caused by anaerobic and some gram positive organisms. Patients develop fever and abdominal pain, with diarrhea often containing leukocytes and blood. Characteristic features of the disease include the formation of inflammatory, yellow tan pseudomembranes in the colon. The diagnosis is made by the C. difficile toxin assay, and the initial treatment of choice is with metronidazole (an alternative is vancomycin). Gastroenteritis is an acute condition, usually caused by a virus or bacterium that presents with vomiting and diarrhea. There is typically no blood in diarrhea associated with gastroenteritis, and most of these infections are self-limited. There is no association with oral antibiotic intake irritable bowel syndrome is predominantly a pain syndrome of unknown etiology in which there is increased frequency of the normal peristaltic and segmentation contractions of the bowel. It is a chronic condition that produces episodic diarrhea with periods of constipation. Salmonella infection is a common cause of food poisoning. Findings include nausea, vomiting, abdominal pain, and diarrhea often with blood.
Ulcerative colitis is a chronic condition that has features of bloody diarrhea, abdominal pain and weight loss. It is limited to the large bowel, and is exclusively a mucosal disease. Ulcerative colitis involves predominantly the rectum, has no skip lesions (mucosa is affected in its entire extension), and does not cause oral or perianal complications. -
Question 42 of 100
42. Question
1 pointsA 33 year old woman gives birth to a 7 lb, 3 oz baby at term after a normal pregnancy. A large, red, raised discoloration of the infant´s face is observed in the delivery room that persists into adulthood. This is likely a component of which syndrome
Correct
Incorrect
Explanation:
The lesion is a port wine stain, a vascular lesion that, unlike the more common strawberry nevus, does not usually regress with age. Port wine stains can be a component of Sturge Weber disease, which can also include similar vascular lesions of the meninges (leptomeningeal angiomatosis) and sometimes cutaneous angiomatosis at other sites. CT of the brain demonstrates a characteristic pattern of hemispheric atrophy thought to be due venous infarction, with tram-track calcification along the cortical ribbon.
Arnold-Chiari malformations are a cluster of related brain and spinal cord malformations in which there is a downward displacement of the cerebellar vermis and tonsils into the foramen magnum. Dandy-Walker malformation includes a distended fourth ventricle with a hypoplastic (or absent) cerebellum.
Neurofibromatosis includes benign and malignant peripheral nerve lesions and café au-lait spots. Tuberous sclerosis includes cortical tubers, adenoma sebaceum of the skin, pancreatic cysts, renal angiomyolipomas, and cardiac rhabdomyomas. -
Question 43 of 100
43. Question
1 pointsA 48 year old male has BP exceeding 165/100mm Hg. Many antihypertensives have been prescribed. But during each of his periodic admissions to the hospital for alcohol detoxification, his BP continues to be elevated. He spends money on alcohol than on medications. What is most likely to be increased in his skeletal muscles?
Correct
Incorrect
Explanation:
Untreated hypertension leads to hypertrophy of arteries and arteriole. The wall to lumen ratio increases as the walls of the blood vessels thicken. It is not entirely clear why arteries and arterioles hypertrophy in hypertensive patients; however, the vessel walls are subjected to extra amounts of stretch at the higher luminal pressures which may stimulate smooth muscle cell growth. Thus the thickened walls of arteries and arterioles in hypertensive patients is an adaption to the hypertension rather than a cause of the hypertension. The increase in blood pressure that occurs during normal exercise can cause the walls of arteries to thicken (and thus the wall to lumen ratio to increase) when the exercise is frequent, even though the normal resting blood pressure is not necessarily elevated in individuals who exercise regularly. [“Wall to lumen ratio” refers to the ratio of wall thickness to lumen radius or to the ratio of the two cross-sectional areas.] Arteriolar density and capillary density are thought to be decreased in hypertensive individuals. The capillaries lack smooth muscle cells in their walls so the wall to lumen ratio of capillaries does not change. Because the density of capillaries and arterioles decreases in hypertensive individuals the total cross-sectional areas of the capillaries and arterioles are also decreased.
-
Question 44 of 100
44. Question
1 pointsInvestigations of a 19 year old man reveal elevated ALT, HBsAg, Anti-HBc, HBeAg, and bilirubin. All other values are normal. The hepatitis B status of this person is which one of the following?
Correct
Incorrect
Explanation:
The presence of elevated ALT, HBsAg, anti-HBc, HBeAg, and bilirubin all point to active hepatitis B. An asymptomatic carrier would not have elevated ALT and bilirubin. The absence of findings on physical examination rules out fulminant hepatitis B. Recovery from acute self limited HBV is associated with the presence of anti-HBs and absence of HBsAg and HBeAg. Someone who is vaccinated against HBV has only anti-HBs in serum. -
Question 45 of 100
45. Question
1 pointsA boy aged 5 years, hospitalized for cellulitis, exhibits global denudation of the skin resulting from splitting of the epidermis at the stratum granulosum. The most likely diagnosis is which one of the following?
Correct
Incorrect
Explanation:
Scalded skin syndrome is a pediatric condition caused by an exfoliative toxin produced by Staphylococcus aureus (which is a common cause of cellulitis). The toxin splits the epidermis at the level of the stratum granulosum, causing global denudation of the skin. Scalded skin syndrome is not associated with suntanning or sunburn. Impetigo is a superficial skin infection, usually caused by group A beta-hemolytic streptococci or staphylococci. Classic clues are eroded pustules covered by honey colored crusts. Impetigo may lead to post-streptococcal glomerulonephritis, a sign of which is red cell casts in the urine. Melasma consists of irregular patches of hyperpigmentation on the face. It most commonly appears during pregnancy and may not completely regress. Tinea corporis is a fungal infection. It is also known as “ringworm” because it presents as an expanding round lesion with an erythematous circinate border. Vitiligo is characterized by irregular patchy depigmentation of the skin that exhibits melanocyte deficiency microscopically. It is possibly autoimmune in origin and maybe related to stress. It is not associated with sun exposure and is a chronic condition.
-
Question 46 of 100
46. Question
1 pointsA 72 year old healthy volunteer enrolls in the study and undergoes a liver biopsy to evaluate the aging process. Which of the following substances will indicate aging at a cellular level in this biopsy
Correct
Incorrect
Explanation:
Lipofuscin is a brown pigment that accumulates with aging. It is believed to be produced from the peroxidation of lipids. Lipofuscin accumulation does not necessarily impair the ability of the cell to function and can be found in the hearts and livers of healthy elderly patients. Beta-carotene is a carotenoid ingested in the diet (found in yellow vegetables such as squash, pumpkins, and carrots) and converted to vitamin A. Excessive beta-carotene can cause a benign yellow orange discoloration of the skin in a condition known as carotenemia. Bilirubin is a pigment derived from the metabolism of the heme group of hemoglobin. As hemoglobin is broken down, it first forms biliverdin, which is subsequently converted to bilirubin. Bilirubin can be conjugated (to glucuronic acid) or unconjugated. The conjugated form (also called the direct reacting portion) accumulates in biliary obstructions. The unconjugated form of bilirubin (indirect reacting) accumulates in hemolytic processes. Hemosiderin is the storage form of iron and stains blue with Prussian blue. Hemosiderin accumulation from breakdown of red cells is seen in chronic passive congestion of the lung (inside hemosiderin-laden macrophages called “heart failure cells”). Hemosiderin deposition is also seen in hemochromatosis, a disorder characterized by abnormal iron storage. Hemochromatosis is seen in patients with increased iron uptake from the gastrointestinal tract and in patients receiving repeated blood transfusion therapy. Melanin is a brown black pigment made by melanocytes in the skin. Melanin is also found in the iris, giving the eye its color. Neuromelanin is a type of melanin found in catecholamine neurons in the brain.
-
Question 47 of 100
47. Question
1 pointsA 62 year old alcoholic male with a history of GERD and coronary artery disease presents with of retrosternal chest pain following an episode of serve retching. It worsens with deep breaths and swallowing. He is tachypneic and tachycardic, and subcutaneous air is palpated around the neck. Which structure is the source of his symptoms?
Correct
Incorrect
Explanation:
This patient has a history consistent with Boerhaave syndrome. In this syndrome, the patient ruptures the esophagus after episodes of retching. The tear usually occurs 3-5 cm above the gastroesophageal junction, and patients display retrosternal chest pain that may radiate to the back. Signs of the disease may include: left pneumothorax, pleural effusion, subcutaneous or mediastinal emphysema, tachypnea, tachycardia, and signs of infection, if more than 24 hours have elapsed since onset. The aorta, left ventricle, lung, and sternum are not involved in the patient´s condition. A history of retching should direct suspicion away from these other structures.
-
Question 48 of 100
48. Question
1 pointsSome members of a family with a hereditary form of emphysema also have hepatic disease. Particular enzyme levels are low, while liver shows accumulation of abnormal from of this enzyme. Increased activity of which serine protease has been most closely related to the emphysema?
Correct
Incorrect
Explanation:
The autosomal recessive disease is α-1-antitrypsin deficiency, in which an abnormal form of α-1-antitrypsin can be produced by, but not released from, liver cells. This enzyme was named when it was found in vivo to function as a good inhibitor of the pancreatic protease trypsin. However, it actually inactivates a wide variety of enzymes with protease activity that have an essential serine residue in the catalytic centers. Examples of these serine proteases include all of the answer choices, and additionally cathepsin G and plasminogen. Surprisingly, α-1-antitrypsin disease specifically targets only the liver and the lungs. The liver disease, which can be mild to life threatening (either due to cirrhosis or, rarely, massive acute hepatic failure), is related to retained α-1-antitrypsin with residual enzymatic activity. The lung disease seems to be the specific result of an inability to turn off elastase produced by neutrophils during lung infections. Neutrophil elastase is one of the few enzymes that can successfully destroy the heavily crosslinked, insoluble, elastin that is presented in the lung interstitium and contributes to its elasticity. With destruction of this lattice like elastic tissue support structure, emphysema can develop in the lungs. Chymotrypsin is a pancreatic enzyme formed in the intestinal lumen that cleaves aromatic aminoacyl bonds. Plasmin is an enzyme involved in fibrinolysis. Thrombin is an enzyme involved in coagulation. Trypsin is a pancreatic enzyme found in the intestinal lumen that cleaves arginyl and lysyl bonds.
-
Question 49 of 100
49. Question
1 pointsA 75 year old type 2 diabetic and hypertensive woman has end stage renal failure has been on dialysis for several years. She fractures her femur. Tissue taken from the fracture shows increased osteoclastic activity, with notable tunnel like dissection by osteoclast into the bony trabeculae. Increased level of which hormone is likely to be responsible for this lesion
Correct
Incorrect
Explanation:
The patient has developed hyperparathyroidism, a well recognized sequela of chronic renal failure. This condition is caused by high levels of serum phosphate and low serum calcium, which stimulate the release of parathyroid hormone (PTH) in an effort to normalize the calcium/phosphate ratio. PTH binding to osteoblasts causes increased expression of RANKL (receptor activator for nuclear factor K ligand), which promotes the differentiation of osteoclast precursors, leading to enhanced bone resorption. This condition, known as renal osteodystrophy, may lead to osteomalacia and osteitis fibrosa cystica, which is classically associated with dissecting osteitis, as described in this question. Calcitonin, which serves to lower serum calcium levels, is produced in small quantities in chronic renal failure, as serum calcium is already pathologically low. Excess cortisol may produce osteoporosis and pathologic fractures, but there is no relationship between chronic renal failure and cortisol excess. Cushing syndrome is generally secondary to adrenal or pituitary adenomas or primary adrenal hyper function. Erythropoietin production is deficient in chronic renal failure. Excess erythropoietin activates erythrocyte precursors but does not activate osteoclasts. Glucagon excess is a very rare entity, occurring in a minority of islet cell tumors, and is not associated with renal failure. High glucagon produces a transitory skin rash, anemia, and a form of diabetes mellitus.
-
Question 50 of 100
50. Question
1 pointsAn infant born at 33 weeks gestation shows signs of respiratory distress at birth. Her mother had gestational diabetes. Lung immaturity in the fetus prior to birth would have been indicated by which one of the following?
Correct
Incorrect
Explanation:
Neonatal respiratory distress syndrome (hyaline membrane disease) occurs in premature infants as the result of a deficiency in pulmonary surfactant (composed primarily of dipalmitoyl lecithin). This is due to inadequate lecithin synthesis by immature type II pneumocytes. It can be diagnosed before birth by analyzing the amniotic fluid: a lecithin:sphingomyelin ratio greater than 2:1 and the presence of phosphatidylglycerol implies lung maturity. The syndrome is the most common cause of death in premature neonates. Risk factors include birth before the 37th week of gestation, birth by cesarean section, and maternal diabetes. Less than 0.5 L of amniotic fluid is the definition of oligohydramnios, which can be associated with renal agenesis. (Failure of the fetal kidneys to pro-duce urine will result in a low amniotic fluid volume.) Alpha-1 antitrypsin deficiency in amniotic fluid is not associated with lung immaturity.
However, deficiency of this enzyme is associated with both emphysema and liver failure in adults. Elevated maternal HbA1c would indicate poor glycemic control in the mother and a potentially increased risk for fetal abnormalities, but it does not serve as direct evidence of lung immaturity. Elevated maternal serum alpha-fetoprotein (AFP) would be expected in fetal neural tube defects. A low maternal AFP is associated with fetal Down syndrome. -
Question 51 of 100
51. Question
1 pointsThe curves shown below show a normal equilibrium point (point A), at which the cardiac output venous return curves intersect, and an additional venous return curve depicted as a dashed line. Which condition can account for the new equilibrium at point B?
Correct
Incorrect
Explanation:
Transfusion of blood increases cardiac output by increasing mean systemic tilling pressure (MSFP) and decreasing the resistance to venous return. The dashed curve in the figure shows a venous return curve that is shifted to the right and rotated up ward. The right ward shift is caused by increasing MSFP from a normalvaIueof+7 to +12 mm Hg. The increase in blood volume distends the blood vessels and thereby decreases the resistance to venous return (which causes the venous return curve to rotate upward). Even with no change in the cardiac output function curve, the cardiac output has increased to 15 L/min, as shown in the figure.
The increase in sympathetic stimulation that accompanies exercise constricts venules and veins and thus increases the “tightness” of the circulation, as reflected by an increase in MSFP. Although the dashed venous return curve shown in the figure could very well be attributed to exercise or sympathetic stimulation, both of these perturbations also shift the cardiac output curve (i.e., cardiac function curve) in an upward direction because they both increase myocardial contractility. Hemorrhage causes a reduction in blood volume which decreases MSFP and thus shifts the venous return curve to the left. Also, the cardiac out put curve is shifted downward when hemorrhagic shock decreases myocardial contractility. Spinal anesthesia blocks the sympathetic nervous system, which lowers MSFP (i.e., the venous return curve shifts to the left) and decreases myocardial contractility (i.e., the cardiac output curve shifts downward). -
Question 52 of 100
52. Question
1 pointsWhich of the following statements regarding the genetic and immunological basis of coeliac disease is correct?
Correct
Incorrect
Explanation:
The prevalence of coeliac disease is 1% in western societies, and is thus one of the commonest immune-mediated diseases. It arises as a result of genetic predisposition, at least 95% of patients are HLA-DQ2 or HLA-DQ8 positive, and also from the specific immune response to the alpha-gliadin component of gluten.
Cow´s milk can produce an immunologically mediated enteropathy, but the condition is rare and transient. The action of tissue transaminases on alpha-gliadin generates epitopes to CD4+ T-lymphocytes, which provoke an inflammatory response in the intestinal wall. In untreated individuals, alpha-gliadin specific CD4+ T cells can be found producing interferon-gamma in the intestinal wall. -
Question 53 of 100
53. Question
1 pointsA 36-year-old man attends clinic with his wife after failing to conceive after 10 years of marriage. Examination reveals that he is tall, thin and has bilateral gynaecomastia. Investigations show high levels of urinary gonadotrophins. What is the most likely diagnosis?
Correct
Incorrect
Explanation:
Gaucher´s and Marfan syndrome do not present with infertility. Noonan´s is associated with short stature. K1inefelter´s is a sex chromosome disorder affecting 1:400 -1:600 male births typically with 47 XXY, XXXYY or XXYY. Andropause is the term for the gradual decrease in serum testosterone concentration with age, but does not occur, usually, until after the age of 50.
-
Question 54 of 100
54. Question
1 pointsMutation in which of the following is associated with Marfan´s syndrome?
Correct
Incorrect
Explanation:
Mutation in the fibrillin-1 gene is felt to be responsible for Marfan syndrome. In Marfan syndrome the lack of normal fibrillin-l lease to overactivity of TGF-b in the wall of the aorta and the heart valves, leading to damage and destruction to the connective tissue which weakens that aortic wall and heart valves, causing them to stretch. SPINK1 mutations are present in pancreatitis. HLA-DR3 mutations are seen in Autoimmune hepatitis, Sjogren syndrome, Diabetes Type 1, SLE, etc. FMR-1 in Fragile X Syndrome and Collagen Type V in Ehlers Danlos syndrome.
-
Question 55 of 100
55. Question
1 pointsA 25-year-old man presents with dark urine, especially in the early morning. Further investigations show that he has haemoglobinuria and haemolyticanaemia. A diagnosis of paroxysmal nocturnal haematuria is made. What is the likely mechanism underlying this condition?
Correct
Incorrect
Explanation:
A 25-year-old man presents with dark urine, especially in the early morning. Further investigations show that he has haemoglobinuria and haemolyticanaemia. A diagnosis of paroxysmal nocturnal haematuria is made. What is the likely mechanism underlying this condition?
-
Question 56 of 100
56. Question
1 pointsWhich of the following statements regarding jejuna biopsy is correct?
Correct
Incorrect
Explanation:
The villus atrophy may be seen with a magnifying glass. Sub-total villus atrophy is seen in a number of conditions other than coeliac disease (that is, severe tropical sprue, cow´s milk / soya sensitivity in children, gastroenteritis, Whipple´s disease, hypogammaglobulinaemia, neomycin therapy, laxative abuse, Norwalk agent). There is a group of patients who present with coeliac disease in older age -sometimes in their 90s. They present with iron deficiency anaemia, osteoporosis or weight loss. -
Question 57 of 100
57. Question
1 pointsA PTT of 42 sec (reference 22 seconds) with a normal PT is NOT suggestive of which one of the following conditions?
Correct
Incorrect
Explanation:
Vitamin K deficiency primarily lengthens the PT and commonly is present with a normal PTT All the other conditions interfere with the PTT-based assessment of clotting but not with the PT-based assessments. -
Question 58 of 100
58. Question
1 pointsWhich one of the following statements regarding discoid lupus erythematosus and systemic lupus erythematosus is true?
Correct
Incorrect
Explanation:
The onset of DLE (discoid LE), or cutaneous lupus erythematosus, is associated with exposure to sunlight and presents with skin lesions on the face, neck, or upper extremities. Only 10% of patients with DLE develop SLE. Many patients with DLE do have positive ANAs, though often in low titers. Unfortunately DLE rarely resolves spontaneously and often produces significant scarring. Systemic immunosuppressive agents are not usually necessary in the management of DLE. -
Question 59 of 100
59. Question
1 pointsOut of the following, which is most associated with a decreased risk of atherosclerosis
Correct
Incorrect
Explanation:
The level of HDL is inversely associated with the risk of atherosclerosis. All of the other states are associated with increased atherogenesis.
-
Question 60 of 100
60. Question
1 pointsThe most common cause of septic arthritis in adults is
Correct
Incorrect
Explanation:
Staphylococcus aureus is the most common cause of septic arthritis in older children and adults, while group B Streptococci and Escherichia coli are important in neonates.
-
Question 61 of 100
61. Question
1 pointsVascular abnormality that may result in bleeding due to immune complex mediated vasculitis is
Correct
Incorrect
Explanation:
Henoch Schonlein purpura is a vasculitis disorder associated with immune complexes.
-
Question 62 of 100
62. Question
1 pointsA 44 year old female with multiple myeloma develops primary amyloidosis. Organ that is LEAST likely to be involved in the development of this disorder is
Correct
Incorrect
Explanation:
Primary amyloidosis, or amyloidosis that occurs as a result of multiple myeloma, is associated with macroglossia (tongue), peripheral neuropathy (nerves), arrhythmias (heart), and skin manifestations. Spleen involvement is seen in secondary amyloidosis.
-
Question 63 of 100
63. Question
1 pointsA 62 year old known alcoholic comes into the emergency room. He has paresthesias in his hands and feet, cannot remember where he lives, but insists that he had a drink with you in the corner bar last week. The man most likely has
Correct
Incorrect
Explanation:
Wernicke Korsakoff´s psychosis is a form of psychosis in which confabulation plays a major role. It is most often associated with chronic alcoholism. It is not observed in encephalitis or meningococcemia. The patient´s paresthesias are consistent with a peripheral neuropathy but are not typical of carpal tunnel syndrome or mononeuritis multiplex, neither of which would cause psychosis.
-
Question 64 of 100
64. Question
1 pointsA 54 year old woman has fatigue, trouble sleeping, and feels “sore all over.” She has no synovitis, but expresses pain upon palpation of multiple parts of her trunk and extremities. Remaining examination is unremarkable. Her rheumatoid factor, antinuclear antibody titers are negative, and her ESR is normal. Her serum analysis is also normal. The most likely diagnosis is
Correct
Incorrect
Explanation:
A 54 year old woman has fatigue, trouble sleeping, and feels “sore all over.” She has no synovitis, but expresses pain upon palpation of multiple parts of her trunk and extremities. Remaining examination is unremarkable. Her rheumatoid factor, antinuclear antibody titers are negative, and her ESR is normal. Her serum analysis is also normal. The most likely diagnosis is
-
Question 65 of 100
65. Question
1 pointsTrue statement regarding infection by the poliovirus is which one of the following?
Correct
Incorrect
Explanation:
Poliovirus is an acute illness produced by three antigenically different strains. Although immunity to each of the three types is life long, infection with one strain does not confer protection against subsequent infection by another strain. Polioviruses are resistant to lipid solvents and stable at a low pH because they lack a lipid envelope. The virus is acquired by the oral route. It subsequently replicates in the oropharynx and lower gastrointestinal tract. Following replication and spread to the local lymphatics, a viremia occurs, with spread to the nervous system. Within the central nervous system, the virus disseminates along neural pathways. Motor neurons are principally affected, primarily in the spinal cord, lower brain stem, and brain stem reticular formation.
-
Question 66 of 100
66. Question
1 pointsA woman aged 55 years presents to her dermatologist because of a persistent rash on the side of her nose. A biopsy reveals basal cell carcinoma. Which one of the following is NOT a risk factor for skin cancer?
Correct
Incorrect
Explanation:
Acne has not been shown to be a risk factor in basal cell or squamous cell skin cancer. Individuals with fair complexions and blond hair are more prone to sunburns and subsequent skin cancer. This is thought to be due to less protection from sunburn due to a lower quantity of melanin pigment in the skin. Chronic UV exposure from the sun is a risk factor for skin cancer. Previous bums or scars are a predisposing factor for skin cancer (called scar carcinomas). Exposure to previous ionizing radiation or inorganic arsenic can be a risk factor for skin cancer.
-
Question 67 of 100
67. Question
1 pointsA 32 year old man presents with jaundice, pruritis, and weight loss. Signs of cirrhosis and portal hypertension are present on examination. His past medical history is significant for ulcerative colitis. The most likely diagnosis is
Correct
Incorrect
Explanation:
Primary sclerosing cholangitis is associated with ulcerative colitis, and this disease is more common in males. Some patients present with cirrhosis; others with hepatosplenomegaly. The combination of cirrhosis and portal hypertension suggests end stage disease. Primary biliary cirrhosis is more common in middle aged women, and there is no association with ulcerative colitis. Jaundice is rare. This patient is too young for cholangiocarcinoma, which is seen mainly in the elderly Patients with cholecystitis present initially with right upper quadrant pain and fever. Acute schistosomiasis presents with intense pruritis and, in some cases portal hypertension and hepatic fibrosis. There is no association with ulcerative colitis.
-
Question 68 of 100
68. Question
1 pointsA 16 year old girl has a painfully enlarged lymph node in her right axilla. Peripheral blood counts are within normal limits. Lymph node biopsy shows numerous granulomas filled with necrotic debris. The most likely infectious agent involved is
Correct
Incorrect
Explanation:
Bartonella henselae is a gram negative rod. Cats and dogs are the reservoir transmitting the organism through bites or scratches. It causes cat scratch fever which presents as regional lymphadenopathy with or without low grade fever and headaches. In the immunocompromised patient, it can cause bacillary angiomatosis, which is the vascular proliferative form of infection and can affect any organ system; although it most commonly affects the skin and subcutaneous tissue. Treatment is with azithromycin or doxycycline. Borrelia burgdorferi is a spirochetal organism that is transmitted by a tick bite (Ixodes spp), producing Lyme disease. Lyme disease progresses from a skin rash to fevers, headache, and pain for about a month. It may produce lymphadenopathy, but is not associated with granuloma formation. Chlamydia psittaci infection occurs after contact with infected bird droppings and produces an atypical pneumonia. The central nervous system may also be involved, but lymph nodes are spared. Chlamydia trachomatis the chlamydial species that typically produces suppurative nodal granulomas (lymphogranuloma venereum). Coxiella burnetii infection is transmitted by inhaling dust or drinking milk from infected mammals, especially sheep and cows. The disease in humans, Q fever, is marked by mild nonspecific symptoms or pneumonia, and may progress to myocarditis or hepatitis. Rickettsia prowazekii produces epidemic (louse borne) typhus, which is transmitted by body lice and produces a rash akin to Rocky Mountain spotted fever. Although the organism (may reside in the lymph nodes in dormancy, it does not elicit granuloma formation. -
Question 69 of 100
69. Question
1 pointsA patient with Epstein-Barr virus (EBV) lymphoma gets himself enrolled in an experimental protocol to stimulate production of CD8+ cells specific for his EBV transformed cells. His peripheral blood cells are tested with a battery of anti-HLA-A, -B, and -C antibodies, followed by addition of complement and trypan blue dye. The wells of the microtiter plate in which dye was taken up by the cells are shown below.
The molecules that are implanted into the membrane of a culture of tissue fibroblasts enriched for expression of costimulatory molecules would serve to stimulate the most effective cytotoxic killing areCorrect
Incorrect
Explanation:
CD8+ cytotoxic T cells are stimulated by the presence of endogenously produced peptides presented in the groove of a class I MHC molecule. In the presence of costimulatory molecules such as B7 and CD28, these killer cells can be stimulated directly in the absence of TH cells. The test performed in the microtiter plate here is a cytotoxicity assay to define the class I haplotype of the cancer patient. Cells exposed to specific anti-class I antisera plus complement will become leaky and take up dye molecules. The haplotype of this patient is HLA-A1/7, HLA-B1/7, and HLA-C2/3, so any of these MHC molecules loaded with peptides from the intracellular virus would stimulate a killing response. The peptides loaded into the groove of the class I molecule must be endogenously synthesized: Whole virus will not be effective. EBNA is a molecule found on the membrane of Epstein-Barr virus transformed cells and would be a good candidate for CTL recognition. Whole EBV by itself would not serve as a stimulus for cytotoxic killing. Small peptides produced during viral infection and transported to the cell membrane in the groove of class I MHC molecules serve as the stimulus for this response. HLA-A1/7 by itself would not serve as a stimulus for cytotoxic killing in this patient. This haplotype is syngeneic for the patient and would not be recognized by itself as foreign. HLA-B2/3 would stimulate a cytotoxic response in this patient, but not one which would be directed against EBV-transformed cells. This haplotype is allogeneic to the patient, so it would stimulate a cytotoxic response, but not the desired one. HLA-C2/3 would not serve as a stimulus for cytotoxic killing in this patient. This haplotype is syngeneic for the patient and would not be recognized as foreign.
-
Question 70 of 100
70. Question
1 pointsA 4 year old girl´s physical examination is remarkable for extremity edema. Urinalysis shows significant proteinuria our no red cells or casts. Electron microscopy performed on a kidney biopsy would most likely reveal which finding?
Correct
Incorrect
Explanation:
The most frequent cause of nephrotic syndrome in children is minimal change disease (Iipoid nephrosis), which is characterized by an absence of findings by light microscopy and by fusion of epithelial toot processes on electron microscopy. Dense deposits are seen in type II membranoproliferative glomerulonephritis. Mesangial deposits are a prominent feature of IgA nephropathy and are seen to a lesser degree in several other glomerulonephritides. Subendothelial deposits are seen in type I membranoproliferative glomerulonephritis. Subepithelial spikes are a feature of membranous glomerulonephritis.
-
Question 71 of 100
71. Question
1 pointsA 67 year old woman dies of severe congestive heart failure. Her past medical history is remarkable for rheumatoid arthritis that first manifested at the age of 35. At autopsy a cross section of her heart had a waxy rubbery texture. The autopsy also reveals also revealed splenomegaly, hepatomegaly, and glomerulopathy. The spleen, liver, and kidneys showed a similar waxy texture. Which mechanism explains these clinical and pathologic findings?
Correct
Incorrect
Explanation:
Chronic inflammatory conditions, such as collagen vascular diseases, rheumatoid arthritis, and bronchiectasis, may lead in time to accumulation of a specific form of amyloid known as AA (amyloid associated protein). Systemic amyloidosis affects many organs, especially the heart, kidneys, spleen, GI tract, liver, and tongue. The ventricular walls become thickened and inelastic, and the resulting pathophysiologic picture is that of restrictive cardiomyopathy. Congestive heart failure ensues because of decreased ventricular compliance and secondary reduced stroke volume. Note how the ventricular lumina are narrowed and the walls are extremely thickened. A waxy texture is highly suggestive of amyloid deposition which can be confirmed histologically by using Congo red staining. Atherosclerosis of coronary arteries may give three different types of clinical manifestations: arrhythmias (with sudden death), angina pectoris, and myocardial infarction. Loss of myocardial tissue due to ischemic necrosis may lead to congestive heart failure. The heart will show evidence of old infarcts in the form of scars, and the ventricular cavities will become dilated.
Coxsackievirus myocarditis is the most common etiologic form of viral myocarditis, which may resolve without clinical consequences or lead to dilated cardiomyopathy and heart failure. Mutations of myosin chain genes are the most common causes of hypertrophic cardiomyopathy. This manifests with thickening of the interventricular septum (so called asymmetric cardiac hypertrophy), with resultant impediment to both systolic outflow and diastolic filling. Here, the ventricular walls are uniformly thickened and the remaining postmortem findings (liver, kidney, and spleen changes) cannot be explained based on a diagnosis of hypertrophic cardiomyopathy. Systemic hypertension in the early stages is associated with cardiac hypertrophy, but progressive ventricular dilatation develops with decompensation. In a patient dying of hypertension related heart failure the heart is usually large (hypertrophic) and the ventricles are markedly dilated. -
Question 72 of 100
72. Question
1 pointsA 54 year old smoker has severe, crushing, substernal chest pain. Coronary arteries are imaged. A complete occlusion of the left anterior descending coronary artery is observed. What best describes the pathophysiologic processes that produced his symptoms?
Correct
Incorrect
Explanation:
Although many of the events that produce atherosclerotic plaques remain elusive, several steps in the process are known. The first gross findings associated with atherosclerotic plaque formation are fatty streaks. Fatly streaks are found in the vasculature of most people in the second decade of life. The fatty streak is a focal accumulation of serum lipoproteins within the intima of the blood vessel wall. As smooth muscle cells migrate to the lesion they produce a connective tissue matrix that overlies the lipid-laden foam cells. When this fibrous covering dislodges, the thrombogenic contents of the lesion are exposed to the circulation and the ensuing clot then occludes the vessels. Tissue ischemia and infarction then occur. The fibrous lesions that follow the fatty streak seldom produce infarction through direct extension of the lesion ischemia may occur with resulting angina, but because the lesion growth is slow, collateral vessels may grow and thus prevent infarction. The fatly streak precedes fibrous lesions found in coronary artery disease. Lipid-laden macrophages, called foam cells, contribute to the core of atherosclerotic lesions. They do not, however, release their lipids through exocytosis, forming a lipid-based occlusion.
-
Question 73 of 100
73. Question
1 pointsAn infant aged 2 months is brought because of a fractured arm. Since birth this is his sixth fracture. On examination he has a soft skull that when palpated, gives the impression of loose bones under the scalp. His sclerae are blue-tinged and he does not respond to sound. What is the likely cause of his problem?
Correct
Incorrect
Explanation:
The patient has osteogenesis imperfecta (“brittle bone disease”), which is a genetic disease that causes very fragile bones. The type illustrated in the question stem is the severe, lethal form; milder forms also exist. The blue sclera may or may not be present. A variety of mutations appear to cause this clinical syndrome, most of which appear to affect type I collagen and the conversion of calcified cartilage to bone. The severe form is often fatal in infancy.
Marfan syndrome causes tall stature, ocular changes, and aortic dissection, and is not usually diagnosed in infancy. Hurler syndrome causes dwarfism and other skeletal changes, and is not usually diagnosed in infancy. Achondroplasia causes dwarfism. You should be aware that milder cases of osteogenesis imperfecta are sometimes misdiagnosed for years as “child abuse”, which can lead to the emotional trauma of the child being removed from the family. -
Question 74 of 100
74. Question
1 pointsA 37 year old woman has cold and painful fingertips, as well as difficulty swallowing and indigestion. Exam shows a thickened, shiny epidermis over the entire body, with restricted movement of the extremities, particularly the fingers, which appear claw like. Which autoantibodies will likely be found in her serum?
Correct
Incorrect
Explanation:
This patient has systemic sclerosis, also called scleroderma. Antibodies to topoisomerase I (anti-Scl-70) occur in up to 70% of patients with diffuse systemic sclerosis, but only rarely in other disorders. Systemic sclerosis, is characterized initially by excessive fibrosis and edema of the skin, especially the hands and fingers, producing sclerodactyly (characteristic changes in the fingers, which resemble claws). Raynaud phenomenon is common. The diffuse type of systemic sclerosis generally spreads to include visceral organs, such as the esophagus (producing dysphagia), the lungs (producing pulmonary fibrosis), the heart (leading to heart failure or arrhythmia), and the kidneys (renal failure causes 50% of scleroderma deaths). Females are affected more than males (3:1 ratio). A more restricted variant of systemic sclerosis, with a somewhat more benign course, is CREST syndrome (Calcinosis, Raynaud syndrome, Esophageal dysmotility, Sclerodactyly, and Telangiectasia), characterized by the presence of anti-centromere antibodies (although 10% of CREST patients will have anti-topoisomerase antibody as well). Anti-ds DNA and anti-Sm (Smith antigen) are characteristic of systemic lupus erythematosus (SLE) but are not common in patients with systemic sclerosis. Rheumatoid factor is an autoantibody directed against IgG. It is found in patients with rheumatoid arthritis. Anti-SS-A is typically seen in Sjögren syndrome (although it may also be seen in SLE).
-
Question 75 of 100
75. Question
1 pointsA 26 year old female has pain and tenosynovitis of the wrists and ankles, and arthralgias of other joints. Two similar episodes have occurred before. She had her menstrual period during the previous week. Exam shows ulcerated lesions overlying the wrists and ankles. These symptoms are likely due to deficiency of
Correct
Incorrect
Explanation:
This patient has disseminated gonococcal infection. Gonococcal arthritis and tenosynovitis typically involve both the upper and lower extremities equally. Vesicular skin lesions are characteristic of disseminated gonococcal disease. Females are at particular risk of gonococcemia during menstruation, since sloughing of the endometrium allows access to the blood supply, necrotic tissue enhances the growth of Neisseria gonorrhoeae, and there is an alteration of the pH. Patients who have a C6-8 deficiency have both an increased risk of gonococcemia and a tendency to have multiple episodes. These patients are also at risk for bacteremia from Neisseria meningitides. Cl esterase inhibitor deficiency can occur as an autosomal dominant disorder or may be acquired. Patients have angioedema without urticaria. The syndrome is also associated with recurrent attacks of colic and episodes of laryngeal edema. Ciliary dysfunction is a marker of Kartagener syndrome (immotile cilia syndrome). The syndrome includes infertility, bronchiectasis, sinusitis, and situs inversus. It is an autosomal recessive disorder caused by abnormalities in the dynein arm of the cilia. Endothelial adhesion molecule deficiency, or beta-2 integrin deficiency, is characterized by failure of neutrophils to express CD18 integrins on their surface. Patients have impaired phagocyte adherence, aggregation, chemotaxis, and phagocytosis of C3b-coated particles. Clinically, there is delayed separation of the umbilical cord, sustained agranulocytosis, recur-rent infections of skin and mucosa, gingivitis, and periodontal disease. Eosinophil deficiency or eosinopenia occurs with stressors such as acute bacterial infection and following administration of glucocorticoids. There is no known adverse effect of eosinopenia.
-
Question 76 of 100
76. Question
1 pointsA 16 year old student ate sweet and sour chicken with vegetables and fried rice at a local Chinese restaurant along with her friends. All the girls developed nausea, vomiting, abdominal pain, and diarrhea within 4 hours of eating lunch. The most likely cause of these symptoms is
Correct
Incorrect
Explanation:
Bacillus cereus produces a self limited diarrhea due to ingestion of the preformed enterotoxin in contaminated fried rice and seafood. The incubation period is typically around 4 hours. The degree of vomiting is greater than the diarrhea. B. cereus is also associated with keratitis, producing a corneal ring abscess. Clostridium botulinum produces a neurotoxin that blocks the release of acetylcholine, resulting in a symmetric descending paralysis that may lead to respiratory complications causing death. Symptoms include blurred vision, photophobia, dysphagia, nausea, vomiting, and dysphonia. Most cases are associated with the ingestion of contaminated home canned food. Clostridium perfringens produces a severe diarrhea with abdominal pain and cramping (sometimes called “church picnic” diarrhea). The incubation period is 8-24 hours after ingesting contaminated meat, meat products, or poultry. The meats have usually been cooked, allowed to cool, and then warmed, which causes germination of the clostridia) spores. EHEC-Enterohemorrhagic Escherichia coli, produces a bloody, noninvasive diarrhea due to the ingestion of verotoxin found in undercooked hamburger at fast food restaurants. The O157:H7 serotype typically produces this syndrome. Some patients develop a life threatening complication called hemolytic-uremic syndrome. Vibrio cholerae typically produces a watery, nonbloody diarrhea with flecks of mucus (rice water stools). Abdominal pain is not a feature. Massive fluid loss and electrolyte imbalance are complications. Patients who ingest contaminated shellfish experience fever, copious watery diarrhea, and abdominal cramps within 48 hours after eating.
-
Question 77 of 100
77. Question
1 pointsA poor Asian patient develops muscle weakness and atrophy with fasciculation and hyporeflexia. His condition continues to deteriorate; eventually his legs are permanently paralyzed. The condition is also observed in several other residents of his town. Which structure in the spinal cord is most likely to be affected by the responsible virus?
Correct
Incorrect
Explanation:
The disease presented in this question is poliomyelitis which is caused by the poliovirus, a picornavirus. The virus is spread via the fecal oral route and can then cause paralysis by infecting the alpha-motor neurons of the anterior horn of the spinal cord. Early symptoms include malaise, headache, fever, nausea, abdominal pain, and sore throat. Later symptoms include flaccid paralysis, muscle atrophy, fasciculations, and areflexia. The central canal has no nerve fibers inside. The corticospinal tract contains upper motor neurons. A lesion of this tract would lead to upper motor neuron symptoms, such as ipsilateral spastic paresis and hyperreflexia. The dorsal columns are a common target of syphilis, a condition known as tabes dorsalis. Dorsal root ganglia contain the neuronal cell bodies of sensory neurons, not motor neurons. The spinothalamic tract has sensory nerve fibers responsible for pain and temperature.
-
Question 78 of 100
78. Question
1 pointsrossly the tips of the renal papillae in a specimen show pale gray discoloration and parenchymal loss. Histologically, there was coagulative necrosis limited to the distal portion of the renal pyramids, without identifiable glomerular or interstitial changes. Which clinical condition would be compatible with this finding?
Correct
Incorrect
Explanation:
The pathologic picture is classic for papillary necrosis. This complication develops most commonly in diabetic patients, but other conditions may result in this renal complication. Among these, analgesic abuse, urinary tract obstruction, and sickle cell disease are the most important. Papillary necrosis may lead to acute renal failure. Fragments of tissue that slough off necrotic papillae may become impacted into the ureters and cause ureteral colic. Gout may cause two renal complications: interstitial nephritis due to deposition of urate crystal within the renal interstitium, and formation of urate calculi within the pelvis. Miliary tuberculosis often involves the kidney. Tubercular infection will give rise to whitish, seedlike nodules most apparent in the renal cortex and consisting of caseating granulomas. Mural thrombosis in the heart frequently leads to systemic thromboembolism. Consequently, embolic infarcts develop in peripheral organs, including kidney, spleen, brain, and the heart itself. In the kidneys, infarcts manfiest grossly as wedge shaped areas of pale coagulative necrosis in the cortex. Recent streptococcal infection may be associated with postinfectious glomerulonephritis (manifesting with nephritic syndrome). The kidneys are edematous and enlarged, and microscopically there is acute glomerulonephritis.
-
Question 79 of 100
79. Question
1 pointsA newborn child born with a congenital infection has excessive growth of new bone on the anterior surface of the tibia with anterior bowing of the bone. This abnormal finding is due to which of the following processes?
Correct
Incorrect
Explanation:
The infant has congenital syphilis. The tibial deformity develops when spirochetes localize in the periosteum, where they produce an active inflammation with many plasma cells. This then causes a massive, reactive, periosteal deposition of bone on the medial and anterior surfaces of the tibia (“saber shin”). “Saddle nose” deformity, another feature of congenital syphilis, is due to inflammatory destruction of the nasal and palatal bones. Gummas, in which a central necrotic focus is surrounded by layers of granulomatous and nonspecific chronic inflammation, can also be found in and near the saber shin and saddle nose. The bone changes are not neoplastic, and involvement of the medulla and cortical layer of the bone is less marked than are the periosteal changes.
-
Question 80 of 100
80. Question
1 pointsA 28 year old female presents with headaches. She has a blood pressure of 180/100 mmHg. Initial investigations reveal a hypokalaemia. On further questioning she is found to consume a large quantity of licquorice. Inhibition of which enzyme is responsible for the pseudohyperaldosteronism associated with Licquorice?
Correct
Incorrect
Explanation:
11β-HSD is responsible for the conversion of cortisol to the inactive cortisone, preventing activation of the mineralocorticoid receptor by cortisol but permitting activation by aldosterone. Both Liquorice and Carbenoxolone inhibit 11β-HSD and produce pseudohyperaldosteronism with hypertension and hypokalaemia yet appropriately low renin and aldosterone concentrations.
-
Question 81 of 100
81. Question
1 pointsWhich one of the following is correct about Leptin?
Correct
Incorrect
Explanation:
Leptin is synthesised within the adipocyte and plasma concentrations are directly related to adipocyte (fat) mass. It acts on centres within the hypothalamus to produce satiety.
-
Question 82 of 100
82. Question
1 pointsWhich of the following would be most in keeping with a diagnosis of polymyalgia rheumatic (PMR)?
Correct
Incorrect
Explanation:
Liver enzymes are elevated in most patients.
Visual disturbances are suggestive of temporal arteritis not PMR, and are due to ischaemic changes in ciliary arteries (optic neuritis/infarction) and less commonly due to central artery occlusion. Raised CK occurs in polymyositis. PMR is rare before the age of 50 years. -
Question 83 of 100
83. Question
1 pointsWhich of the following is hepatitis D associated with?
Correct
Incorrect
Explanation:
The hepatitis D virus, or delta agent, is a defective RNA virus which cannot replicate except in the presence of hepatitis B virus. Consequently, it occurs as a co-infection or, in the presence of chronic HBV infections, as a superinfection. Hepatitis C can lead to chronic hepatitis in 30-40% of cases. It is usually transmitted via blood transfusions and intravenous needles. It is not associated with hepatitis D. Hepatitis A rarely becomes chronic or aggressive. Hepatitis A is transmitted via the fecal-oral route. Alcoholic hepatitis is associated with the same symptoms as presented in this patient, plus right upper quadrant pain and several other symptoms that make it difficult to distinguish alcoholic hepatitis from extrahepatic biliary obstruction. Hepatitis D Virus is irrelevant to Schistosomiasis.
-
Question 84 of 100
84. Question
1 pointsWhich one of the following findings would be most useful in differentiating between TTP and HUS?
Correct
Incorrect
Explanation:
Microangiopathic hemolytic anemia, thrombocytopenia, elevated serum LDH levels, and normal coagulation test are present in both thrombotic thrombocytopenia purpura and hemolytic-uremic syndrome. However, neurologic abnormalities are typically present only in TTP not HUS.
-
Question 85 of 100
85. Question
1 pointsA 28 years old female presents with complaints of vaginal bleeding and severe nausea and vomiting. During the pelvic examination, she is found to have an enlarged uterus. She has been trying to get pregnant for the past 15 months, with no success. She is now excited however, because a home pregnancy test was positive. An ultrasound demonstrates a multi-loculated cystic structure within the uterine cavity, but there is no clearly defined embryo. Serum hCG levels are markedly elevated (>100,000 mlU/ml). The mass is removed with dilation and curettage of the uterus after establishing the diagnosis and although placenta like tissue is observed during pathologic examination, fetal tissue is not present. Which of the following karyotypes are revealed on chromosomal examination?
Correct
Incorrect
Explanation:
The lesion described is a hydatidiform mole, of which two forms exist, complete and partial. In a complete mole, the karyotype is 46, XX (or, in a few cases, 46, XY). In this form, fetal tissue is not identified. The most common presenting symptom is vaginal bleeding. Serum hCG levels are dramatically increased and the uterus is enlarged. Hyperemesis can occur in patients with complete moles, probably due to the extremely high hCG levels, but is rare in patients with partial moles. The chorionic villi have a characteristic grapelike, multicystic appearance. It is important to note that unlike the partial mole, which very rarely develops into choriocarcinoma, complete moles carry a 2% increased risk for the development of choriocarcinomas.
This represents a postfertilization event with either a diploid or triploid genotype rather than, haploid (choice A). The disease is unrelated to Turner syndrome, XO (choice B)
A complete mole contains the normal number of chromosomes, and does not contain an additional single Y chromosome (choice D)
In a partial hydatidiform mole, fetal tissue is often evident. The conceptus is triploid, with a 69, XXX or 69, XXY karyotype (choice E). Both enlarged and normal villi are present. -
Question 86 of 100
86. Question
1 pointsThe most common form of Klinefelter´s syndrome is due to which of the following?
Correct
Incorrect
Explanation:
Klinefelter´s syndrome is most often due to the 47, XXY karyotype. It is not the result of low testosterone synthesis (A), testosterone resistance (B), 45Y/47, XXY mosaicism (D), or 47, XXX/47, XXY mosaicism (E).
-
Question 87 of 100
87. Question
1 pointsOut of the following, which ovarian carcinoma is associated with an elevated human chorionic gonadotropin (hCG) level?
Correct
Incorrect
Explanation:
CA-125 may be elevated in ovarian epithelial tumors, such as a cystadenocarcinoma. AFP can be elevated with either an endodermal sinus tumor or embryonal carcinoma. An elevated hCG can be seen with choriocarcinomas. Sertoli-Leydig and granulosa-theca cell tumors do not usually produce an elevated CA- 125, AFP, or hCG level.
-
Question 88 of 100
88. Question
1 pointsA 38 year old black male complains of dark urine and fatigue. He has never had this before. However, he has begun to take primaquine for prophylactic anti-malarial protection for his upcoming trip to Africa. The most likely diagnosis is
Correct
Incorrect
Explanation:
Glucose 6 phosphate dehydrogenase deficiency is an X linked disease present in approximately 15% of all black males in America. It is typically unsymptomatic until challenged with a drug agent which may cause hemolysis, such as primaquine, quinidine, or sulfonamides.
-
Question 89 of 100
89. Question
1 pointsAll of the following are considered as risk factors for coronary artery disease, EXCEPT
Correct
Incorrect
Explanation:
Some studies have suggested that minimal consumption of alcohol might actually be beneficial in reducing atherosclerosis. Cigarette smoking, hypertension, elevated serum cholesterol levels, and a family history of coronary artery disease are all well described risk factors
-
Question 90 of 100
90. Question
1 pointsA man of 38 years age presents with arthritis of the wrists. Roentengram studies reveal chrondrocalcinosis (linear calcifications) on the articular cartilage surface. The most likely diagnosis is
Correct
Incorrect
Explanation:
The diagnosis of pseudogout is often mistaken with other disease states, such as hyperparathyroidism, hypothyroidism, hemochromatosis, and hypomagnesemia. However, the presence of linear calcifications on the surface of articular cartilage in affected arthritic joints is only characteristic of pseudogout, or calcium pyrophosphate deposition disease.
-
Question 91 of 100
91. Question
1 pointsA germline B lymphocyte is determined to possess 200 V gene segments, 20 D gene segments, and 50 J gene segments to encode heavy-chain variable domains. It also possesses 100 V gene segments and 10 J gene segments to encode light-chain variable domains. How many distinct idiotypes can be produced in the absence of recombinational inaccuracies?
Correct
Incorrect
Explanation:
During their development, progenitor B and T lymphocytes undergo complex gene segment rearrangements that allow them to create a wide diversity of antigen recognition molecules. In the case of a B lymphocyte, three gene segments (V, D, and J) are randomly recombined to create the coding for the variable domain of the heavy chain of immunoglobulin. Two gene segments (V and J) are randomly recombined to create the coding for the variable domain of the light chain of immunoglobulin. Analogous rearrangements are made in T cells to create the variability of the N-terminal domains of the T-cell receptor beta or alpha chains (respectively). In order to determine the maximum number of distinct idiotypes (antigen combining sites) in this question, you simply multiply the different possible choices: for the heavy chain, 200 x20 x50, or 2 x 10^5 possibilities; for the light chain, 100 x 10, or 1 x 10^3 possibilities. Since heavy and light chains randomly associate following rearrangement, the product of the two possibilities (2 x10^5) x (1 x10^3), or 2 x 10^8 gives the total number of possible combinations. Option A, 1,000 is not correct. This is the number of distinct light-chain variable domains that could be produced. Option B 4000, is not correct. This is the product of they and D gene segment rearrangements only for the heavy chain. Option C, 1 x 10^4, is not correct. There is no way to arrive at this figure by multiplying combinations of VDJ gene segments. Option D, 2 x 10^5, is not correct. This is the total number of heavy-chain variable domains that could be produced, but an immunoglobulin idiotype consists of both heavy and light-chain variable domains.
-
Question 92 of 100
92. Question
1 pointsA 33 year old medical technician has a history of acute eczematous dermatitis on her hands and wrist in the distribution of the latex gloves she wore. Her hands skin is dry, crusted, and thickened. The reaction cleared after a vacation of 2 weeks. After 72 hours back on the job, the eczematous dermatitis returned and continued to grow worse. What characterizes the technician´s reaction to the latex gloves?
Correct
Incorrect
Explanation:
Sensitization to latex has become a major health-care problem. Local skin irritations are common but more severe allergic reactions occur, up to and including rare anaphylactic reactions that are occasionally fatal. The immune responses to latex are immediate-type hypersensitivity (type I) reactions, expressing themselves in minutes, and/or delayed-type hypersensitivity (type IV) reactions, which will express themselves in 48 to 72 hours. The type I reactions are due to IgE-mediated sensitivity to latex proteins, whereas type IV reactions are due to a cell-mediated response to the chemicals that are added in the processing of latex. The type IV response in this circumstance would be referred to as contact dermatitis. Irritant dermatitis can be observed in the early stages of sensitization and can be due to sweating, rubbing, and residual soap. In the present case, the timing of the reaction after 72 hours points to the type IV reaction rather than an irritant dermatitis. A type I reaction would have been apparent in minutes with characteristic rhinitis, conjunctivitis, urticaria, asthma, angioedema, or anaphylaxis after the technician put on her latex gloves. A type II reaction is a cytolytic response mediated by an antibody. The type II reaction is observed in the context of hemolytic anemias, thrombocytopenia, neutropenia, etc. A type III reaction is an immune complex response that sets into motion an inflammatory response.
-
Question 93 of 100
93. Question
1 pointsA 25 year old male has itching on his arms and face. Examination shows well circumscribed wheals with raised, erythematous borders and blanched centers. The type of hypersensitivity that is this patient most likely having is
Correct
Incorrect
Explanation:
Urticaria (hives) is a good example of a local anaphylaxis reaction which is classified as a type I hypersensitivity reaction. Type I hypersensitivity reactions involve preformed IgE antibody bound to mast cells or basophils, which release vasoactive and spasmogenic substances when they react with antigens. The eosinophil-mediated cytotoxicity against parasites is an example of antibody-dependent, cell-mediated cytotoxicity. Type II hypersensitivity is caused by autoantibodies against cells or tissues. When these antibodies activate complement or promote phagocytosis, the tissue is damaged. Type III hypersensitivity is caused by immune complex deposition in the small diameter blood vessels and the subsequent activation of complement and destruction of the vasculature. The tuberculin (PPD) reaction, used to test for tuberculosis exposure, is an example of delayed-type, or type IV, hypersensitivity.
-
Question 94 of 100
94. Question
1 pointsA neonate´s family history is strong for birth defects. Chromosomal analysis of fetal cells does not demonstrate any abnormalities. Immediately at birth, cells are extracted from the umbilical cord and sent for flow cytometric analysis. Analysis shows a population of cells that are both CD4+ and CD8+. These cells are which one of the following?
Correct
Incorrect
Explanation:
CD4 and CD8 are both markers for T cells, and mature T cells usually express one or the other but not both. However some of the immature T cells in the thymus (that can “leak” into the baby´s blood in small numbers) coexpress these markers, before the cells have “committed” to being either helper T or cytotoxic T cells. In adults, significant numbers of cells coexpressing CD4 and CD8 usually implies the existence of a non-Hodgkin lymphoma with a cell population similar to immature T cells. Normally T-cell precursors entering the thymus are double positive for CD4 and CD8. During thymic selection, cells that recognize MHC class II become CD4-positive only and differentiate into helper T lymphocytes; whereas cells that recognize MHC class I become CD8 positive only and differentiate into cytotoxic T lymphocytes.
Helper T cells are CD4+ and CD8-, suppressor T cells are CD4- and CD8+. Immature B cells and plasma cells will be both CD4- and CD8-, since these are T-cell markers. -
Question 95 of 100
95. Question
1 pointsA 44 year old man with diabetes insipidus, severe polydipsia, and polyuria is admitted for surgical repair of an inguinal hernia. Following surgery, he exhibits fever and psychic disturbances. His plasma sodium concentration is 175 mEq/L. Figures A-E below show the relative osmolarity (Y-axis) and volume (X-axis) of the intracellular and extracellular fluid compartments during normal conditions (solid line) and following various disturbances in the body fluids (shaded area, dashed line). Which figure most accurately depicts his condition after surgery?
Correct
Incorrect
Explanation:
Diagram D shows a disturbance in body fluid balance referred to as “hypertonic contraction” which is characteristic of loss of hypotonic fluid from the body. Loss of hypotonic fluid decreases total body water and increases body fluid osmolarity, as indicated in this diagram. Normal function of the thirst center ensures that polyuria (excessive urine output) closely matches polydipsia (excessive water intake) so that dehydration does not occur. However, when adequate replenishment of water loss by excretion is interfered with (i.e., during surgical procedures), dehydration may become severe, causing fever, psychic disturbances, and even death. The plasma sodium concentration of 175 mEq/L in this patient is a clear indication that he is dehydrated and that serum osmolarity is elevated (extracellular osmolarity can be approximated as 2 times the plasma sodium concentration, which is 350 mOsm/L). Option A (isotonic contraction) can be caused by loss of isotonic fluid, e.g., acute diarrhea. Option B (hypertonic expansion) can be caused by excessive intake of sodium chloride without the drinking) of water. Option C (hypotonic contraction) is characteristic of sodium chloride loss from the body, e.g., secondary to lack of aldosterone.
Option E (hypotonic expansion) can be caused by retention of water by the kidneys, e.g., inappropriate secretion of antidiuretic hormone. -
Question 96 of 100
96. Question
1 pointsMagnetic resonance image of a 46 year old woman shows marked degeneration of the caudate nucleus. Based on her neuroimaging studies, which would be the most likely symptoms observed in this patient?
Correct
Incorrect
Explanation:
This patient most likely has Huntington disease. The caudate degenerates markedly in this disease although the putamen and nucleus accumbens also degenerate. Atrophy of the caudate and putamen can make the lateral ventricles appear very large when imaged. Bradykinesia, resting tremor, and difficulty initiating movements are typical of Parkinson disease, which results from midbrain substantia nigra degeneration. Intention tremor and akinesia are signs of cerebellar dysfunction. Moon facies, amenorrhea, and hypertension are typical of Cushing syndrome. Additional symptoms include truncal obesity, increased facial hair, acne, irritability, depression, and decreased energy. This can be caused by an ACTH-secreting pituitary adenoma. Motor and vocal tics are indicative of Tourette syndrome.
-
Question 97 of 100
97. Question
1 pointsA 70 year old male has had polycythemia year for 12 years and was treated by phlebotomy alone, and has remained stable. He has noted a dragging sensation in his abdomen recently, and physical examination reveals massive splenomegaly, palpable to 7 cm below the ribs. This finding may represent the conversion of polycythemia vera to which hematologic disorder?
Correct
Incorrect
Explanation:
There are four types of myeloproliferative disorders: chronic myeloid leukemia (CML), polycythemia vera, myeloid metaplasia with myelofibrosis (MMM), and essential thrombocythemia (which is very rare). Both CML and polycythemia vera are characterized by hypercellular bone marrow producing all three cell lines. Over a period of years, the bone marrow may burn out, and the marrow space is replaced by fibrosis. In this circumstance, hematopoiesis (including production of neoplastic cells) moves to extramedullary sites, predominantly the spleen and liver. This process is called myeloid metaplasia with myelofibrosis. Polycythemia vera does not involve the lymphoid cell line, so progression to lymphoid malignancies such as acute lymphoblastic leukemia, chronic lymphocytic leukemia, multiple myeloma or Waldenstrom macroglobulinemia is unlikely. Although both polycythemia vera and CML can progress to MMM polycythemia vera does not progress to CML. Hairy cell leukemia a leukemia often associated with massive splenomegaly, is derived from a B lymphocyte, not a myeloid stem cell. It would not progress to a myeloproliferative disorder.
-
Question 98 of 100
98. Question
1 pointsA 12 year old child fractures her right radius and ulna in an accident. Her arm is placed in a cast extending over the elbow, which nearly eliminates movement of her forearm muscles. After several weeks in the cast what will occur in her forearms?
Correct
Incorrect
Explanation:
The primary function of blood vessels in skeletal muscles is to supply nutrients and remove metabolic waste products from the muscle tissues. Prolonged increases in metabolic rate lead to growth of the vasculature; decreases in metabolic rate lead to vascular rarefaction, i.e., loss of capillaries as well as other types of blood vessels. In other words, blood vessels grow where they are needed and regress when they are not needed. When a limb is immobilized in a cast, the muscles are inactive and their metabolic rate decreases. This decrease in metabolic rate leads to a decrease in capillarity in the skeletal muscles by mechanisms that are poorly understood. When the metabolic rate of immobilized skeletal muscle decreases, the oxygen consumption of the muscle decreases, which is likely to cause an initial increase in both myoglobin oxygen saturation and intracellular pO2. Vascular endothelial growth factor (VEGF) is thought to stimulate the angiogenesis caused by exercise training. Immobilizing skeletal muscle would be expected to decrease VEGF production.
-
Question 99 of 100
99. Question
1 pointsA 30 year old woman develops visual impairment. Over the past several weeks her peripheral vision has progressively deterioriated. The problem has now become quite serious. On exam she has a loss of peripheral vision; however, her remaining vision is 20/20. She also has intermittent headaches. CT scan shows a large mass in the seller region. Which systemic symptom would most likely to be present?
Correct
Incorrect
Explanation:
Pituitary adenomas are benign neoplasms of the anterior lobe, sometimes associated with excess hormone secretion. Most, about 70% of adenomas, are nonfunctioning. Depending; on which cell type is affected, different clinical syndromes may be present. Adenomas may occur at any age; they affect both sexes, but most commonly affect men between the ages of 20 and 50. Overall they are rare and may have an association with MEN I (pituitary adenoma, parathyroid neoplasm, pancreatic islet adenoma). Prolactinomas are the most common type of pituitary adenomas, resulting from neoplastic growth of the lactotroph cells in the anterior pituitary. Young women with prolactinomas present clinically with amenorrhea galactorrhea, and infertility. Men usually have impotence and decreased libido. Acromegaly is generally caused by a functional adenoma secreting excess growth hormone. This disorder occurs after the epiphyses of the long bones have fused and causes thickening and increased coarseness of the bones and skin. Systemic manifestations include arthritis, insulin resistance cardiomegaly, and sexual dysfunction. Cushing syndrome is the result of a corticottroph adenoma producing excess ACTH which stimulates adrenal corticosteroid hypersecretion. It is less common than prolactinoma or null cell (no hormone) adenoma. Cushing disease refers to ACTH dependent hyperadrenalism, whereas Cushing syndrome refers to hypercortisolism from any cause, Obesity, hypertrichosis, and amenorrhea are the usual clinical results. Neurogenic diabetes insipidus is due to disease of the posterior pituitary leading to vasopressin (antidiuretic hormone; ADH) deficiency and is not associated with pituitary adenoma. Although the etiology is not clear in approximately one third of cases, others are associated with craniopharyngiomas. The disorder is characterized by polyuria, thirst, and polydipsia. Virilization is often associated with gonadotroph adenoma (uncommon). Clinically, it causes acquired hypogonadism in men, as well as headache and visual disturbances. -
Question 100 of 100
100. Question
1 pointsA male aged 50 year old man complaining of chest pain radiating down his left arm is admitted to the emergency department. Exam reveals pulmonary edema and effusions. His medical history is significant for hyperlipidemia and diabetes. Selected pressures are shown below. The most likely diagnosis is
Pulmonary artery pressure: 22/7 mm Hg Pulmonary wedge pressure: 10 mm Fig Left ventricular pressure: 170/5 mm Hg Aortic pressure: 120/80 rum Hg Correct
Incorrect
Explanation:
This patient has aortic stenosis. In aortic stenosis, the blood is ejected from the left ventricle through a smaller-than-normal opening. Because the resistance to ejection of blood is high, the left ventricular pressure can sometimes increase to over 250 mm Hg with normal systolic pressure in the aorta. Note that in this patient, the left ventricular peak systolic pressure has increased to 170mm Hg, the aortic systolic pressure is normal at 120 mm Hg, and a 50mm Hg gradient exists across the aortic valve during systole. In aortic regurgitation, blood flows backward through the aortic valve during diastole when the valve is closed. Left ventricular systolic pressure and aortic systolic pressure are nearly the same with pure aortic regurgitation. Mitral regurgitation means there is backward flow of blood through the mitral valve during systole. This accumulation of extra blood in the left atrium during ventricular systole leads to an elevation in the pulmonary wedge pressure (which is used as an estimate of left atrial pressure). In mitral stenosis, blood must flow from the left atrium into the left ventricle through a smaller-than-normal opening. Because the resistance to blood flow through the mitral valve is higher than normal, the left atrial pressure (estimated by the pulmonary wedge pressure) is often several mm Hg higher than the left ventricular end diastolic pressure. Right-sided heart failure is most commonly due to left-sided heart failure: blood simply backs up through the lungs and into the right heart. However, this patient has not yet developed right heart failure. All of the pressure recordings in the right heart and lungs are normal. Note i that if one undergoes cardiac catheterization, only arterial pressures (left-sideds can be obtained. One must access the venous-side for right-sided pressures. Pulmonic regurgitation would result in a high wedge pressure.